Sie sind auf Seite 1von 82

Ctethelper.blogspot.

com

Ctethelper.blogspot.com
Real Number

Number System
Lets say you have some apples and if somebody asks how many apples you have, what
will you answer? One got to put a "label" on them. And here comes "numbers" to help
you out. We need numbers to define the quantity and to perform mathematical
operations like addition and subtraction. Numbers can classify in various depending on
what they represent and where they are used. Here they are:

Natural Number
And how will counting start if you are counting your apples? You have to start with 1 and
then go on. (One doesn't have to count, if they have no apple to begin with).

Number line is a line where we can represent the numbers and their fractional or
decimal parts. A number lines start from 0 to either sides, both positive and negative
and goes on till positive and negative infinity.
Real Numbers are those numbers that can be represented on a number line.
Natural Numbers, Whole Numbers, Integers all belongs to Real Numbers.

Decimal Number
Decimal Numbers have an integer plus a decimal part or the fractional part.
For example, the number 15.02 is a decimal number having 15 as the integral part and
the .02 as the fractional part.

Natural Numbers are those numbers that starts from 1, followed by 2,3 and so on.
Natural Numbers= {1, 2,3,4,5...}

Irrational Numbers
Irrational Numbers are the numbers that cannot be expressed on the number line.
These numbers can't be expressed as a terminating decimal.

Whole Number

For example, 2, 2, 3 are some of the irrational numbers.

Whole numbers are natural numbers with the inclusion of zero. Whole numbers starts
from Zero, followed by 1, 2 and so on.

Negative Numbers

Whole Numbers = {0, 1,2,3,4...}


or Whole Numbers = 0 + Natural Numbers.

Negative numbers have the same absolute value as a positive numbers, but with a
different sign. For Example the absolute value of -5 and 5 is 5 but their sign is opposite.

Integers

One can't see negative numbers in nature. One can observe that there are 5 cows in the
field but one cannot say that there are -5 cows in the field. Negative numbers are used to
represent opposing quantities. For example credit and debit, positive charge and
negative charge.

Integers are whole numbers plus the inclusion of negatives numbers from -1, -2, -3, and
so on. Integers = Whole Numbers + { -1, -2, -3, -4, -5,...}

For example, a credit $5 and debit of $5 adds to 0 because their absolute value is same
but of opposite nature.
5 unit positive charge and 5 units of negative charge add to zero.

Ctethelper.blogspot.com

Ctethelper.blogspot.com
Example: Predecessor of number 45 is (45 - 1)= 44 and the successor of 45 is (45 +1)
=46.

Prime Numbers
Prime number are those numbers that have only two divisors - the number 1 and the
number itself. Example: 1,2,3,5,7,11... are some prime numbers. Try to divide these
numbers using various divisors and you will know that they are only divided by the
number 1 and the dividend itself.
Special case of number 1: The number 1 is not a prime number. The
smallest prime number is the number 2 not the number 1.

The second number will be read as five crore three lakh twenty one thousand six
hundred fifty four.

Addition (+)
Commutative Property

We take a number 2 and added it to another number 8, the result is 10. Now, We take a
number 8 and added it to another number 2, the result is again 10.

Composite Numbers
Any number that is not a prime number is a composite number.Example of composite
numbers are 1,4,6,8,9,10,12,14,15,16,18,20,...These numbers have more than two
divisors.

This property of addition, where addition of two numbers is always equal, irrespective of
the order of addition is known as commutative property of addition.
a + b = b + c, where a and b are any two numbers.
Associative Property

Place Value
Place value is calculated by multiplying the digit whose place value is to be calculated
and the place (hundredth, thousandth etc...). For example: if we have to calculate the
place value of 5 in the number 858412. Then first we have to decide the place at which 5
is located. Here we can see that 5 have 4 digits on the right side. Therefore we have to
multiply 5 by 10000 to get the place value. ( 5 x 10000 = 50000).

We take a number 5 and added it to another number 9, the result is 14 and add the
result of previous addition to 8 to arrive at 22. Now, We take a number 9 and add it to
number 8 and the result of this addition is added to 5, the result is again 22.
This property of addition, where addition of a group of number is always the same
irrespective of order of the addition is known as associative property of Addition.
(a + b) + c = a + (b + c) , where a,b and c are numbers.

An easier method is to discard all the digits before the digit whose place value we are
calculating. For the above number we will discard 8 and we will left with 58412. Now
replace all digits after the digit 5 to get the place value for the required digit. i.e. 58412
=> 50000.

Subtraction (-)
We have,

Predecessor and successor

8-3=5

Predecessor of a number X is a number that comes 1 count before the number X. To get
predecessor of number X we have to subtract the number X by1. In the same way
successor of a number X is number that immediately follows the number X

and
3-8=-5
Therefore, Subtraction is not a commutative.

Ctethelper.blogspot.com

Ctethelper.blogspot.com

Multiplication ( x )

Solved Problems On Number System

We have,

Q1: In the following, which is the greatest number ?

8 X 3 = 24
and

(1). (4)

3 X 8 = 24

(2). (222)

Therefore, Multiplication is commutative.

(3). [(2 + 2)]

And

(4). (2 + 2 + 2)

(8 X 3) X 5 = 120
and

Answer: (3). [(2 + 2)]

3 X (8 X 5) = 120

Explanation:

Therefore, Multiplication is also associative.

1. (4)

4x4=

2. (222) = (8) = 8 X 8

16
=

64

Division ()

3. [(2 + 2)] = [(4)] = [16]=16 x16 = 256 ( and hence this the answer)

We have,

4. (2 + 2 + 2) = (6) =

36

84=2
and

Q2. 407928 is read as

4 8 = 0.5

(1) Four lakh seventy nine thousand twenty eight

Therefore, Division is not a commutative and obviously, not associative.

(2) Forty seven thousand nine hundred twenty eight


(3) Forty thousand nine hundred twenty eight
(4) Four lakh seven thousand nine hundred twenty eight
Answer: (4) Four lakh seven thousand nine hundred twenty eight

Ctethelper.blogspot.com

Ctethelper.blogspot.com

Q3.If an operator is defined as

Explanation:

43=4+5+6

Don't get scared by "Hindu-Arabic Numeration" argon. Now to get at the answer just
subtracted the lowest 4 digit number from the highest 4 digit number ( and add + 1
since both the ranges are included ) to arrive at the answer.

54=5+6+7+8

i.e. 9999 - 1000 + 1 = 8999 + 1 = 9000

64=6+7+8+9
What will n 8 be equal to ?

Q5. The number 49532 rounded off to the nearest thousand is

(1) n + 28

(1) 49000

(2) 8n + 28

(2) 49500

(3) 8n + 36

(3) 41000

(4) n + 36

(4) 50000

Answer :(2) 8n + 28
Explanation: From the examples given in the question, the first number is the starting
number of the series and the second number represents the number of numbers in the
series inclusive the starting number of the series.
Using the above rule, we can expand n 8 to n
+(n+1)+(n+2)+(n+3)+(n+4)+(n+5)+(n+6)+(n+7)

Answer: (4) 50000


Explanation: The thousand part in the given number is 49 thousand. Now we will look
at the rest of the number i.e. 532 which is greater than 500 and hence can be rounded
off to 1000.
Therefore, the answer is 49,000+1000 = 50000

= 8n + (1 + 2 + 3 +4 +5 +6 +7)
= 8n +28
Q6. When faced with word problems, Rajan usually asks should I add or subtract?
Should I multiply or divide?. Such questions suggest
Q4. How many 4-digit numbers are there in the Hindu-Arabic Numeration System ?

(1) Rajan seeks opportunities to disturb the class

(1) 99

(2) Rajan has problems in comprehending language

(2) 8999

(3) Rajan lacks understanding of number operations

(3) 9999

(4) Rajan cannot add and multiply

(4) 9000

Answer :(2) Rajan has problems in comprehending language

Answer :( 4) 9000

Ctethelper.blogspot.com

Ctethelper.blogspot.com

Explanation: From his words it is clear that he is confused about using the
mathematical operators.

i.e. 5671 - 1765 = 3906

Q9.Study the following pattern:


1X1=1
Q7. Sum of place value of 6 in 63606 is
11 x 11 = 121
(1) 6606
111 x 111 = 12321
(2) 6066
.... .... What is 11111 X 11111?
(3) 18
(1) 12345421
(4) 60606
(2) 123453421
Answer: (4) 60606
(3) 1234321
Explanation:
(4) 123454321
The number is 63606. Place value of first six in the number, from left to right, is
60000, place value of second six is 600 and last six is 6.
Adding place values of all the sixes we have,

Answer :( 4) 123454321

60000 + 600 + 6 = 60606

Explanation: From the pattern given in examples, we can observe that:


1. The numbers of digit in the answer = Sum of number of digits in each multiplier - 1.

Q8. The difference of 5671 and the number obtained on reversing its digits is

2. The resulting number have digits starting from 1 to digit having value equal to the
number of digits in either number and then decreases from there to 1.

(1) 3916
(2) 7436

i.e. 111 X 111 = 12321. The Digits in the answer start from 1 and goes to 3 ( Number of
digits in 111) and decreases from there till digit 1 .

(3) 3906

Applying the same rules to 11111 X 11111, we know the right answer is 123454321.

(4) 4906
Q10 which of the following is correct?
Answer :( 3) 3906
Explanation: The number is 5671 and the number obtained by reversing the digits is
1765.

(1) Predecessor of predecessor of 1000 is 999

Ctethelper.blogspot.com

Ctethelper.blogspot.com

(2) Successor of predecessor of 1000 is 1001

Let put a plus sign on the incoming toys and negative sign on all outgoing toys
(negative signs because outgoing decreases the inventory). Then, we have 239 + 70 =
152

(3) Successor of predecessor of 1000 is 1002


(4) Predecessor of successor of 1000 is 1000

Answer: (4) predecessor of sucessor of 1000 is 1000

Q12.In the product 3759 x 9573, the sum of tens' digit and units' digit is

Explanation:

(1) 9

1. Predecessor of predecessor of 1000 is 999 => predecessor of (1000 - 1 )=> 1000- 1 - 1


= 998 999.Rejected

(2) 16
(3) 0

2. Successor of predecessor of 1000 => successor of (1000 - 1) => 1000 - 1 + 1 => 1000
1001 Rejected!
3. Successor of predecessor of 1000 => successor of (1000 - 1 ) => 1000 - 1 + 1 => 1000
1002 Rejected!
4. Predecessor of successor of 1000 => predecessor of (1000 + 1) => 1000 + 1 -1 =>
1000 = 1000 Success!

(4) 7
Answer: (4) 7
Explanation: Just multiply the last two digits from each number and you will get the
tens' digit and the ones digit of the resultant. Add the digit to get the answer.

Q13. 19thousands + 19 hundreds + 19 ones is equal to


(1) 20919
Q11.A shop has 239 toys. Seventy more toys were brought in.Then 152 of them were
sold. The number of toys left was

(2) 19919

(1) 239 - 70 + 152

(3) 191919

(2) 239 + 70 -152

(4) 21090

(3) 239 -70 -152

Answer :(1) 20919

(4) 239 +70 +152

Explanation: We can take one thousand out 19 hundreds as 19 hundreds makes 1


thousand and 900 hundred. Add 1 to 19,000 to get 20,000. Then add remaining 900
and 19 to get the answer, 20919.easy!!

Answer: 239 + 70 -152


Q14. If 567567567 is divided by 567, the quotient is
Explanations:

Ctethelper.blogspot.com

Ctethelper.blogspot.com

(1) 10101
(2) 1001001

Answer: (1)55

(3) 3

Solution:

(4) 111

Cost of 1 pencil = 2.5


Cost of one and half dozen ( 1 dozen = 12) or 18 pencil = 2.5 x 18 = 45

Answer: 1001001
Explanation: The first impulse is to tick 111. nope. Just do it in your head or on piece of
paper. your will get 1 quotient for first 567 and then you have to add 0 and 0 to
quotient then 1 to clear 567 and then again 0 and 0 and then 1.

Since he gave 100 rupee note to the shop keeper, he will get ( 100 - 45 ) = 55 rupees from the
him.
Q17. When 90707 is divided by 9, the remainder is
(1)3

Q15.How many 1/8 are there in 1/2?

(2)5

(1) 16

(3)6

(2)8

(4)7

(3)4

Answer: (2) 5

(4)2

Solution:

Answer: (3) 4

Divide the 90707 by 9 and you will get 5 as remainder and 10078 as quotient.

Solution: Divide 1/2 by 1/8 to get the answer.


Therefore, No. of 1/8 in 1/2 = (1/2)/(1/8) = (1/2)x(8) = 4

16. A pencil costs two and half rupees. Amit buys one and half dozen pencils and gives
hundred rupee note to the shop keeper.The money he will get back is
(1)55
(2)45
(3)65
(4)30

Quicker Method: Try Rule of divisibility for 9. According to this rule if the sum of
the all the digits of the dividend is divisible by 9 then the number itself is divisible by 9.
Here,
Sum of digits of 90707 = 9 + 7 + 7 = 23
and if subtract 5 from this number we will get 23 - 5 = 18 therefore it should be the
remainder of the division.

Ctethelper.blogspot.com

Ctethelper.blogspot.com

Q18. The sum of the place value and the face value of the number 3 in 12345 is

Answer :(2)123456

(1) 0

Solution:

(2) 295

Looking at the pattern, it is evident that the answer will have :

(3) 297

(a) 6 digits because there are six digit in the number beginning with 98.. and digit 6 is
subtract from it

(4) 305
(b) Starting from 1 and ending on 6
Answer: (3) 297
Therefore the number is 123456
Solution:
Place value of 3 in the number 12345 is 3 X 100 = 300

Q 20 The numbers of integers less than -3 and greater than -8 are :

And the face value of 3 is 3 itself.

(1) 2

Then the difference = 300 - 3 =297

(2) 3
(3) 4

Q19. Look at the following pattern:


(9 - 1) / 8 = 1
(98 - 2) / 8 = 12
(987 - 3) / 8 = 123
(9876 - 4) / 8 = 1234

(4) 6
Answer: (3) 4
Solution:
Numbers of integers greater than -8 and less than -3 are -7,-6,-5,-4. And hence 4 in
numbers.

..
(987654 - 6) / 8 =

(1) 12345
(2)123456
(3)123465
(4) 123467

Q21. The value of 0.001 + 1.01 + 0.11 is


(1) 1.111

(2) 1.101

(3) 1.013

(4) 1.121

Answer: (4) 1.121


Solution:

Ctethelper.blogspot.com

Ctethelper.blogspot.com

Aligning the decimal point of all the number given in the problem , we have

Taking first equation, a^3 = 1 + 7 = 8


a^3 = 2^3

0.001
=> a = 2

1.010

Taking second equation, 3^3 = 1 + 7+ b

0.110

27 = 8 + b
1.121
19 = b
Taking third equation, 4^3 =1 + 7 + c
Q22. In 1999, the population of a country was 30.3 million. The number which is the
same as 30.3 million is

64 = 8 + c
56 = c

(1) 303000000

(2) 30300000

(3) 3030000

(4) 3030000000

Therefore value of a,b and c came out to be 2,19 and56 respectively.

Answer:

(2) 30300000

Solution:
1 Million = 1000000

And a + b +c = 56 + 2 + 19 = 77
Q 24: We call a number perfect if it is the sum of all its positive divisors , except itself.
For example 28 is a perfect number because 28 = 1 + 2 + 4 + 7 +14. Which of the
following is a perfect number?

Therefore, 30.3 million = 30.3 x 1000000 = 30300000


___________________________________________________________
Q23. If a^3 = 1 + 7 , 3^3 = 1 + 7+ b and 4^3 =1 + 7 + c, where a, b and c are different
positive integers, then the value of a + b + c is

(1) 13

(2) 10

(3) 9

(4) 6

Answer: (4) 6
(1) 58

(2) 68

(3) 77

(4) 79

Answer:
Solution:

Solution:
(1) Factors of 13 other than itself are 1. Not this number.
(2) Factors of 10 other than itself are 1, 2, 5 and their sum = 1 + 2 + 5 = 8. Not this
number.
(3) Factors of 9 other than itself are 1, 3. Not this number.

Ctethelper.blogspot.com

Ctethelper.blogspot.com

(4) Factors of 6 other than itself are 1,2,3 and their sum = 1 + 2 + 3 = 6.This is number.
Both 16^/3^500 and 2/3^500 are negligible values and therefore can

____________________________________________________________

be ignored.

Q25. The product of two whole numbers is 24. The smallest possible sum of these
numbers is

What's left is = ( 3^2 - 1) / 1 = 9 - 1 = 8


____________________________________________________________

(1) 8

(2) 9

(3) 10

(4)12

Q27. If 800880 = 8 x 10^x + 8x10^y + 8x10^z where x, y and z are whole numbers,
then the value of x + y + z is

Answer: (3) 10
Solution:
First let find all pairs of number that can give 24 upon multiplication. They are ( 8, 3)
,(12,2),(6,4) and upon addition they give out 11, 14 and 10 respectively. 10 is the
smallest.

(1) 11

(2)8

(3) 6

(4) 15

Answer:
Solution:

____________________________________________________________
800880 can be broken down into
Q26. The value of

800880 = 800000 + 800 + 80


( 3^502 - 3^500 + 16 ) /( 3^500 + 2) is
= 8x10^5 + 8x10^2 + 8x10^1
Comparing it with the equation given in the question, we have

(1) 2

(2) 4

(3) 8

(4) 16

x =5, y =2 and z=1


And

x+y+z=5+2+1=8

___________________________________________________________

Answer: (3) 8
Solution:
Dividing both numerator and denominator by 3^500, we get
{ ( 3^2 - 1) + 16/3^500} / (1 + 2/3^500)

Q28. The number n is doubled and then y is added to it.The result is then divided by 2
and the original number n is subtracted from it. The final result is
(1) y

(2) y/2

(3) n + y

(4) (n + y )/2

Answer: (2) y/2

Ctethelper.blogspot.com

Ctethelper.blogspot.com

Solution:
And since we got b after subtracting a from 14 , this means a + b =14
Going step by step
____________________________________________________________
1. Taking a number

=> n

2. Double the number

=> 2n
Q30. Which of the following is not a perfect square:

3. Add y

=> 2n + y

4. Result is divided by 2
5. Subtract n

=> n + y/2
=> y/2

____________________________________________________________

(1)548543213

(2)548543251

(3)548543215

(4)548543241

Q 29: If
1957
-

a9
18b8

Answer: (4)548543241
Solution:
None of the perfect square will have 3 at the one's digit. This rules out option (1).
Again only a perfect square of a number that has digit 5 at ones place will have 5 at the
unit digit but never alone. It will have 2 in the tenth place. This rules out option (3).

Then sum of a and b is


(1) 15

(2)14

(3)13

(4)12

Try the division method for any one of the option (2) and (4) and you will get that only
option (4) is a perfect square.
____________________________________________________________

Answer: (2)14
Solution:
Starting from the right hand side of the subtraction. Since 7 is less than 9, 1 has to be
borrowed from 5. Again 9 is reduced to 8 in the answer, therefore 1 has to borrowed
from 9.This means 5 becomes 14 after borrowing from 9 and giving 1 to 7.

Q31. When an integer K is divided by 3, the remainder is 1, and when K + 1 is divided by


5, the remainder is 0. Of the following, a possible value of K is
(a) 62 (b) 63
(c) 64 (d) 65
Answer:
Solution:

Ctethelper.blogspot.com

Ctethelper.blogspot.com

Using Divisibility Rules, We know that any number that is divided by 5 has to have
either 0 or 5 in the unit place. Therefore, (K+1) has 5 in the unit place and K should have
either 4 or 9 in the unit place. While checking for options only option (c) has a number 4
in the unit place and hence the CORRECT answer.

=>

Q32. The least prime number is

Q34. If {1/2(a-b)} + ab = p(a + b), then the value of p is (assume ab)

(a) 1

(b) 3

(c) 2

(d) 0

x + y = 1 --------(iv)

Now, (x - y) = (x-y)(x+y) = (7)(1) = 7

(a) 1/2

(b) 1/4

(c) 1/8

(d) 1

Answer: (c) 2
Solution:

Answer:

2 is the lowest prime number and also 2 is the only even number.

Solution:
Taking LHS

Q33. If 999x + 888y =1332 and 888x +999y =555, then x - y is equal to
(a) 7

(b) 8

(c) 9

(d) 5

{1/2(a-b)} + ab = 1/4(a-b) + ab
= 1/4(a + b- 2ab) + ab
= 1/4((a + b) -ab/2 + ab

Answer :( a) 7

= 1/4(a + b) + ab/2

Solution:

=1/4(a + b + 2ab)

999x + 888y =1332 ----------(i)


888x + 999y = 555 ----------(ii)
Subtracting (ii) from (i)
=> 111x - 111y = 777
=> (x - y) = 7 ----------(iii)
And adding (i) and (ii)
=> 1887x + 1887y = 1887

= 1/4(a+b)
Thus the equation in the question becomes 1/4(a+b) = p(a + b)
Therefore. p = 1/4

Ctethelper.blogspot.com
HCF and LCM
Factors and multiple : If a number a divides number b exactly, we say
that a is a factor of b in this case , b in called a multiple of a .

Highest common factor (H.C.F) or greatest common measure (G. C.M.)


or greatest common Divisor (G.C.D.). The H.C.F. of two or more than two
numbers is the greatest number that divides each of them exactly.

Ctethelper.blogspot.com
Dividing 54 by 24 we will get 2 as quotient and 6 as remainder.
Now divide the divisor 24 by the remainder 6. We will get 0 remainder. Since
the number 6 was the last divisor, we have 6 as the H.C.F.

Finding the HCF of more than two number: Suppose we have to find the
HCF of three numbers. Then, HCF, of { HCF of any two and the third
number} gives the HCF of three given numbers.
Similarly, the HCF of more than three number is obtained.

There are two methods of finding the HCF of a given set of numbers :
1. Factorizing method : Express each one of the given number as the
product of prime factors. The product of least of common prime factors gives
HCF.
Example: Lets say we have to find the HCF of 24 and 54.

Least Common Multiple( LCM): The least number which is exactly


divisible each one of the given numbers is called their LCM.
1 Factorization method of Finding L.C.M : Resolve each one of the given
numbers in to a P roduct of Prime factors then , L .C. M is the Product of
highest Product of all the factors

Lets prime factorized the number 24 and 54.


24 = 2 * 2 * 2 * 3
54 = 2 * 3 * 3 * 3

only 2 *3 common in both the numbers.

2.Common Division Method (Short-cut Method ) of finding L.C.M :


Arrange the given numbers in a row in any order : Divide by a number which
divides exactly at divisible . Repeat the above Process til no of the numbers
are divisible by the same number except 1. the Product of the divisors and
the undivided numbers is the required L.c.M of the given numbers
4. Product of two numbers = Product of their H.C.F.. and L.C.M.

Therefore, 6 is the HCF of 24 and 54.


5. Co-primes : Two numbers are said to be co-primes if their H.C.F. is 1.
2.Long Division Method: Suppose we have to find the HCF of two given
numbers. Divide the larger number by the smaller opne. Now divide the
larger number by the smaller number one.; Now, Divide the divisor by the
remainder. Repeat the process of dividing the preceding number by the
remainder last obtained till zero is obtained as remainder. the last divisor is
required HCF.
Taking above numbers 24 and 54.

6. H.C.F and L.C .M of Fractions :


HCF= ( HCF . of Numbers / LCM. of Denominators ) 2. LCM.= (of
Numbers | HCF of Denominators

Ctethelper.blogspot.com

Ctethelper.blogspot.com
2. The ratio of the two numbers is 3:4 and their LCM is 180. The second number is

7. HCF and LCM of Decimal Fractions : in given numbers , make the


same number of decimal places by annexing zeros in some numbers, if
necessary. Considering these numbers without decimal Point , find HCF or
LCM as the case may be Now , in the result, mark off many decimal Places
as are there in each of the given numbers,

(a) 45

(b) 90

(c) 30

(d) 60

Answer: (d) 60
8,Comparison of Fractions : Find the LCM of the denominators of the
given fractions. Convert each of the fractions into an equivalent fraction with
LCM as the denominator , by multiplying both the numerator and
denominator by the same number the resultant fraction with the greatest
numerator is the greatest
Solved Questions:

Solution:
The point to understand here is that,
When we find ratio of two numbers then the HCF of the numbers is eliminated while
reducing the ratio to the lowest term. Thus, the two numbers are 3 x HCF and 4 x HCF
Now, Let the HCF be x. Then using the formula
LCM x HCF = First Number and Second Number

Q1. Find the Least Number which when divided by 12,18,36 and 45 leaves
the remainder 8,14,32 and 41 respectively.

180 x HCF = (3 x HCF)(4 x HCF)

(a) 180

(b) 178

=> 180/(3x4) = HCF

(c) 186

(d) 176

=>

Answer: (d) 176

15 = HCF

Therefore, the second number is 4 x15 = 60

Solution:
First we have to find the relationship between the Divisors and the
remainder.
We see that 12-8=4 , 18-14 = 4, 36-32=4 and 45-41 = 4

3. A farmer has 945 cows and 2475 sheep. He farms then into flocks,
keeping cows and sheep separate and having the same number of animals in
each flock. If these flocks are as large as possible, then the maximum
number of animals in each flock and total number of flocks required for the
purpose are respectively

The rule here is to find the LCM of 12,18,36 and 45. That is 180.
Now, the required number is obtained by subtracting the common difference
(4) obtained earlier from the LCM of the number.
i.e. 180 - 4 = 176

(a) 15 and 228


(c) 45 and 76

Answer:(c) 45 and 76
Solution:

(b) 9 and 380


(d) 46 and 75

Ctethelper.blogspot.com

Ctethelper.blogspot.com

Here we just have to find the Highest Common Factor of these two numbers
(945,2475). It will come out to be 45. Therefore, looking at the options the
CORRECT answer is option (c).

PROFIT AND LOSS

4. If x:y be the ratio of two whole numbers and z be their h.c.f. then the
l.c.m. of those two numbers is
(a) yz

(b) xz/y

(c)

(d) xyz

xy/z

Cost price: Cost price is the price at which the seller has acquired the
article in question.

Selling Price: Selling price is the price at which the seller is willing to sell
the product.

Profit: Profit is the amount the seller earns by selling an article at a price
greater than the cost price.
Answer:(d) xyz
Solution:

Profit = Selling Price - Cost price

The main point to note here is that when we find ratio of two numbers then
the what is reduced from them is the h.c.f. of both the numbers.
Thus the two numbers are (x.z ) and (y.z)
Again, using the formula,

Loss: Loss is the amount the seller loses by selling an article at a price less
than the cost price.
Loss = Cost price - Selling Price

Multiplication of two numbers = (l.c.m.)(h.c.f)


=>
=>

(x.z)(y.z) = (z)(l.c.m.)
l.c.m.

= xyz

Profit Percentage: Profit percentage is the profit percent that the seller
earns on the sell the article. It is calculated on the cost Price.

Overheads: overheads are the expenses that the seller has to incur to sell
the products/articles. Overhead includes the rent of the shop, salaries of the
employees, utilities bills, advertisement bill or any kind of expenses the
seller has to pay to keep his business running.

Fixed Costs: Fixed cost is the cost that the seller has to bear in spite of
quantity he is selling. Fixed cost include the rent , utility bills, employee
salary and so

Ctethelper.blogspot.com
Variable Costs: Variable cost or direct cost is the cost the seller has to bear
to sell the article. The cost price of the articles, cost price of the raw material
and others.

Ctethelper.blogspot.com
7. If an article is sold at a profit of x% which is further sold at a profit of y%
then
the final Selling price is

Formula for profit and loss:


1.Profit Percentage = {(S.P. - C.P)/C.P.}x100

Final Selling Price = Initial Cost


Price{(100+x)/100}{(100+y)/100)

2.Loss Percentage = {(C.P. - S.P.)/C.P.}x100

8. If two successive profits x and y are made on the same article then the
net

3.To find the Cost Price when Selling Price (S.P.) and Percentage profit (x) is
given
Cost Price =

{S.P/(100 + x)} x 100

4.To find the Selling Price when the cost price (C.P.) and percentage profit
(x) is

profit is
(x + y + xy/100)
Note: above formula could also be used for loss. Just make sure to use
+ve
sign for profit and -ve sign for the loss.
9. If a false weight is used instead of the true weight then, the profit

given by
Selling Price = {(100 +
x)(C.P.)/100}

percentage is
{(True Weight - false weight)/False Weight} x 100

5.To find the Cost Price when Selling Price (S.P.) and Percentage loss (x) is
given
Cost Price =

{S.P/(100 - x)} x 100


Solved Examples:

6.To find the Selling Price when the cost price (C.P.) and percentage loss
(x) is

1. An article is sold for 75 at a loss of 25%. Then find the cost price of the
article.

given by
Selling Price = {(100 - x)(C.P.)/100}

Solution:

Ctethelper.blogspot.com
Using the formula 5 mentioned above we have ,
Cost Price =

{S.P/(100 - x)} x 100

{75/(100 - 25)} x 100

{75/75}x 100

(1/1)x100

Ctethelper.blogspot.com
Net profit/loss = ( 15 - 15 -225/100)
= (-2.25)
First, A made a profit of 15% and again total loss of (-2.25)% in the entire
deal was profit to A.
Therefore, total Profit A made = 15+2.25 = 17.25 %

100
4. I gain 0.70 paisa on Rs. 70. My gain percent is :

2. An article with a cost price of Rs. 840 is sold for a profit of 10% and then
again sold for a profit of 20%. Find the final selling price.

Solution:

(a)0.1%

(b)1%

(c)7%

(d) 10%

Solution:

Using the formula 7 mentioned above, we have


Final Selling price = 840{(100+10)/100}{(100+20)/100}
= 840(110/100)(120/100)
= 840(11/10)(12/10)
= 840(1.1)(1.2)
= 1108.80
3. A sells a mobile to B worth Rs. 10,000. He makes a profit of 15%. Again
B
sold it back to A at a loss of 15%. Find the A's profit or loss.
Solution:
Using the formula 8, taking Y as negative, We have

Total profit = 0.70 paisa


And C.P. = Rs 70
Therefore, Gain percent = (0.70/70)x100 = 1%

5. In terms of percentage profit , which is the best transaction ?


C.P (in Rs.)

profit (in Rs.)

(a)36

17

(b)50

24

(c)40
(d) 60
Solution:

19
29

Ctethelper.blogspot.com

Ctethelper.blogspot.com

We have to calculate the percentage profit in each case, the table becomes

Solution:
Let the Cost price was 100 then the selling price at 40% profit will be 140.

C.P (in Rs.)

profit (in Rs.)

profit %

(a)36

17

47.22

(b)50

24

48

(c)40

19

47.5

(d) 60

29

48.33

Therefore, the best transaction is (d)

And at this price a discount of 10% is offered then the final selling price will
be
140 - (0.10)(140) = 140 - 14 = 126
And net profit is

126 - 100 = 26 and in percentage it will be 26%

8. Profit after selling a commodity for 425 is the same as the loss after
selling it for 355.The cost of the commodity is
(a) 390

(b) 395

(c) 400

(d) 385

6. A shopkeeper purchased 70 kg of potatoes for Rs.420 and sold the whole


lot at the rate of Rs. 6.50 per kg. What will be his gain percent ?

Answer:(a) 390

(a) 4(1/6) %

(b)6(1/4)

Solution:

(c)8(1/3)%

(d)20%

Since the profit after selling the commodity at 425 and loss after selling it at
355 is the same.Then the Cost Price for the commodity lies exactly in the
middle of the two numbers. It will be found by finding the averages of the
these two numbers.

Solution:
Rate per kg at which the potatoes are bought = 420/70 = Rs 6 per kg

There the Cost Price

= (425 + 355)/2 = 390

And they are sold at 6.5 kg per kg.


Therefore total profit = (0.5/6)x100 = 8(1/3) %
7. The price of the a radio is raised by 40% above the cost price and sold at
a discount of 10%. What will be percentage of profit?

9. A watch is purchased for 400 and sold for 460.The profit percentage is
(a) 15.5%

(b) 12%

(c) 13%

(d) 15%

(a) 14%

(b) 30%

Answer:

(c) 25%

(d) 26%

Solution:

Answer: (d) 26%

Total profit = 60 and cost price is 400. Therefore the percentage profit is

Ctethelper.blogspot.com

Ctethelper.blogspot.com

(60/400)x100 = 15 %
Answer:
10. By selling an article for 450 , a man loses 10%. The gain or loss percent
if he sells it for 540 is
(a) gain 9%

(b) loss 9%

(c) gain 8%

(d) loss 8%

Solution:
Let the Selling price be 100. Then the discount of 10% is 10. The selling
price
will become 90. Again a 20% discount is offered then the selling price
becomes 90 - (0.2)(90) = 72.

Answer: (a) gain 9%

Hence the net discount becomes 100 - 72 = 28%

Solution:
Here, Cost Price = 450(110/100) = 495
If he sold it for 540, the profit would have been (540 -495) =

(b) 28%

45

13. A dealer marks his goods at 40% above the cost price and allows a
discount of 20% on the marked price. The dealer has a
(a) loss of 20% (b) gain of 25%
(c) loss of 12% (d) gain of 12%

And therefore the profit = {(45)/(495)} x100 = 9%


Answer:(d) gain of 12%
11. The cost price : selling price of an article is a : b. If b is 200% of a then
the percentage of profit on cost price is
(a) 75% (b) 125%
(c) 100% (d) 200%
Answer:(c) 100%

Solution:
Let the Cost price be 100. Then after it was marked up 40% , the selling
price would be 100 + (0.40)(100) = 140.
Again a discount of 20% was offered on the marked price then the final
selling price become

Solution:

140 - (0.20)(140) = 140 - 28 = 112

Given that b is 200% of a. This implies that if a = 1 then b = 2.

And the profit is 112 - 100 = 12% or gain of 12%

This Implies cost price : Selling price = 1:2


And profit percentage = {(2-1)/1}x 100 = 100%
12. The successive discounts of 10% and 20% are equivalent to a single
discount of
(a) 30%
(c) 25%

(b) 28%
(d) 27%

14. A person sells 400 mangoes at the cost price of 320 mangoes. His
percentage of loss is
(a) 10
(c) 20

(b) 15
(d) 25

Answer: (c) 20 %
Solution:

Ctethelper.blogspot.com

Ctethelper.blogspot.com

Out of 400 mangoes he sold,he was able to recover the cost price of 320
mangoes. And his lost is the cost price of the 80 mangoes.

Compound Interest

Thus, total loss = {(80)/400} x 100 = 20 %

Compound interest: At the end of the interest cycle, if the interest is getting added to the
sum, initially borrowed, to create a new principal, then this kind of arrangement is
called as compound interest.

15. A shoe company sold 50 pairs of shoes on a day costing Rs. 189.50
each

In compound interest for the new cycle , interest is calculated on the sum borrowed
plus on the accumulated interest.
Formula for calculating compound interest

for Rs. 10,000. Then the profit obtained in rupees is


(a) 522

(b) 525

(c) 573

(d) 612

Amount = P { 1 + (r/100)}t
Where,

Answer:
Solution:

P = Principal Amount or the amount that was initially lend

Cost of 50 pairs of shoes = (189.5)(50) = 9475


And he sold them for

r = rate of interest per annum at the principal was borrowed

10000

Therefore , total profit = 10000 - 9475

t = time period ( in years ) for which the amount was


borrowed

= 525

Some Important Formulas:


1. If interest is compounded Quarterly

If the interest is compounded quarterly,


then,

Ctethelper.blogspot.com

Ctethelper.blogspot.com

the time 't' in years becomes (t/3) as there are 4 quarters in a year of 3 month each

Solution:

and the rate of interest r becomes (r/3).

Using the formula used above, we have

And the formula for compound interest becomes


2178 = 1800(1 + 10/100)t

Amount = P [ 1 + {(r/3)/100}]t/3

=>

1.21 = (1.1)t

=> t = 2

Because 1.12= 1.21

2. If interest is compounded Monthly


If the interest is compounded monthly,
then,
the time 't' in years becomes (t/12) as there are 12 months in a year

Q2. The difference between the simple and compound interest on a certain
sum of money for 2 years at 4% per annum is Rs.1. Find the sum.
(a) Rs .630
(b) Rs. 620
(c) Rs. 625
(d) Rs. 635

and the rate of interest r becomes (r/12).


And the formula for compound interest becomes

Answer:(c) Rs. 625


Solution:

Amount = P [ 1 + {(r/12)/100}]t/12

Let the number be X then


Simple Interest at the end of 2nd year = (X x 4 x 2)/100 = ( X x 8)/100 =
(2X)/25
Compound Interest at the end 2nd year = A - P = X {(1 + 4/100)2 1 }= X(0.0816)

Q1. The compound Interest on Rs. 1,800 at 10% per annum for a certain
period of time is 378. Find the time in years
(a) 3 years
(c) 2 years

Again it is given that the difference between the two types of interest is Re 1
Therefore,

(b) 2.5 years


(d) 2.8 years

Answer:(c) 2 years

0.0.0816X - (2X)/25 = 1
=>

X(0.0816 -0.08 )

=>

X(0.0016) =1

=> X = 1/0..0016 = 625

=1

Ctethelper.blogspot.com

Ctethelper.blogspot.com

Simple Interest:

Formula for calculating Amount:

When a borrower takes money from a lender then the borrower has to pay some
interest to the lender for using that money. If the interest does not get added to principal
amount at the end of the payment term, then this kind of arrangement is called as
simple Interest.

Amount = Simple Interest + Principal

Terms associated with Simple Interest:

Few Solved Problems in Simple Interest:


Principal

Term
quarterly,

The total Money borrowed.

The Time period of calculation of interest. May be


half -yearly or yearly.

Rate of interest:

Amount
the

Rate of interest is the rate at the the money is lent.

Amount is the sum of principal and interest to be paid at

Problem 1:
A sum of Money will double it self in 10 years at simple rate of interest. Find the rate of
Interest?

Solution:

It is given to us that a certain sum of money will double itself in 10 years. We have to
find the rate at which it was lent.

end of the term.


Let the principal be P
The the Amount after 10 years would be 2P
Formula for calculating Simple Interest:
Using the formula for amount ,
we have,

Simple Interest = (P x R x T) /100

Ctethelper.blogspot.com

Ctethelper.blogspot.com

Amount = SI + Principal

S.I. = (P x R x T)/100

=> 2P = (PxRx10)/100 + P ( dividing both LHS and RHS by P)


=>

2 = R/10 + 1

Data given in the question,

=>

1 = R/10

R = 14% per annum

=>

R = 10 %

T = 5 years

Thus, any kind of money lent at simple rate of interest of 10% will double itself in 10
years.

S.I. = 330
P =?

Put all the data in the above equation, we get


330 = (P x 11 x 5)/100
=> 330 = (Px11)/20
=> P = (330 x 20)/11 = 30x20 = 600

Problem 2:

Hence , a sum of Rs. 600 will earn a simple interest of 330 in 5 years at 11% rate of
interest.

The Simple interest earned on a certain sum of money in 5 years at 11 % per annum is Rs
330. Find The sum.

Solution:

The equation for simple interest is :


Problem 3:

Ctethelper.blogspot.com

Ctethelper.blogspot.com
years. Now again using the new value of rate of interest ( 12 + 3 = 15) to get simple
interest:

If 600 amounts to 816 in 3 years at simple interest. If the rate of interest is increased by
3%, to how much would it amount to in the same time period?

S.I. = (600x15x3)/100 = ( 6 x 15 x 3) = 270


And amount would have been = 600 + 270 = 870.

Solution:

The equation for simple interest is :

S.I. = (P x R x T)/100

Data given in the question,

R=?
T = 3 years
S.I. = 816 - 600 = 216
P = 600

Put all the data in the above equation, we get


216 = (600 x R x3)/100
=> 216 = 6 x R x 3
=>

R = 216/18 =12 %

Now, the question was if it is to be increased by 3% then what will be the amount in 3

Ctethelper.blogspot.com

Ctethelper.blogspot.com
(c) 1050

(d) 370

RATIO AND PROPORTION


Answer:
1. If A:B = 3:4 B:C =8: 10 and C:D =15:17 then find A:B.C.D

(b) 250

Solution:
In such question, the first we add all the part. we get 4 + 5 + 6 = 15. Out these, B got 5
parts.

(a) 9:12:15:17

(b) 6:12:20:32

(c) 9:22:32:34

Thus, B got (5/15) x 750 = 250

(d) 7:19:27:42
4. Two number are in the ratio of 9: 14 if the larger number is 55 more than the smaller
number.Find the numbers.

Answer: (a) 9:12:15:17

(a) 99,154

(b) 48:88

Solution:

(c) 17:96

(d) 59:74

If there are three ratio a:b and c:d and e:f


Then the ratio among these 4 quantities is given by ace: bce : bde : bdf

Answer: (a) 99,154

Using this the required ratio is:

Solution:

3 x 8 x 15:4 x 8 x 15: 4 x 10 x 15 : 4 x 10 x 17
= 9 : 12 : 15 : 17

The smaller number be x, then the larger number be (x+ 55) . Then according to the
question
x / (x+55) = 9/14

2. Find the ratio compounded of the four ration:


4:3, 9:13, 26:5, and 2:15

Thus,
14x = 9(x + 55)
(x+55)/14 }
=> 14x = 9x + 495

Solution:
The ratio compounded = (4 x 9 x 26 x 2)/(3 x 13 x 5 x 15) = 16/25

=> 5x = 495
=>

3. An amount of Rs 750 is distributed among A,B and C in the ratio of 4:5:6 what is the
share of B ?
(a) 500

( multiplying the LHS and RHS of the above equation by

(b) 250

x = 99

Thus the number are 55 and 154.

Ctethelper.blogspot.com

Ctethelper.blogspot.com
=>

5x = 3(x + 20)

5.The sum of the three is 98 if the ratio between the first and the second be 2:3 and that
between the second and the third be 5:8, then find the second number .

=>

5x = 3x + 60

(a) 90

(b) 45

=>

2x = 60

(c) 77

(d)30

=>

x = 30

And ( x +20) = 30 + 20 = 50
Answer: (d)30
And therefore the sum is 50 + 30 = Rs . 80
Solution:
Let the three numbers be A , B and C.
Then the ratio of A to B is

= 2:3 or 10:15

And the ratio of B to C is =

5:8

15:24

Thus, the ratio of A:B:C is 10:15:24.

7. The ratio between two number is 3:4 If each number be increased by 6 the ratio
becomes 4:5 find the number.
(a) 43,16

(b) 18,24

(c) 22,12

(d) 32, 22

Again, Given that A + B + C = 98.


Therefore, A = (10/49) x 98 = 20
B = (15/49) x 98 = 30

Answer: (b) 18,24


Solution:
Let the number be 3x and 4x. Then

6. A sum of money is divided between two people in the ratio of 3:5. If the share of one
person is Rs 20 less than that of other,find the sum .
(a) 88 Rs

(b) 76 Rs

(c) 85 Rs

(d) 80 Rs

Again, (3x+6/4x+6) = 4/5 ( given that when both numbers are increased by 6 the ratio
becomes 4/5)
=> 5(3x + 6 ) = 4(4x + 6)
=> 15x + 30 = 16x + 24
=> 15x - 16x = 24 - 30

Answer: (d) 80 Rs
=>

-x = -6

=>

x =6

Solution:
Let the share of one person be x then the share of other person is x + 20.
Thus, the first number is 6 x 3 = 18
It is given that
and the seconder number is 4 x 6 = 24
(x / x +20) = 3:5

(3x/4x) = 3/4

Ctethelper.blogspot.com

Ctethelper.blogspot.com
Solution:

8. The ratio between two number is 3: 4. If each number be increased by 2, the ratio
becomes 7:9. Find the number.

Let the number be 2x,3x and 5x.


It is given that if 20 students in each class are increased then the ratio becomes 4:5:7

(a) 43, 16

(b) 22,10

(c) 12, 16

(d) 18,9

It can written as (2x +20) : (3x +20) : (5x + 20) = 4 : 5 : 7


Taking the first two ratio
we have,
Answer: (c) 12, 16
(2x +20) / (3x +20) = 4/5
Solution:
Let the number be 3x and 4x. Then

=>

5(2x +20) ) = 4(3x +20)

=>

10x + 100 = 12x + 80

=>

10x - 12x = 80 - 100

(3x/4x) = 3/4

Again, (3x + 2/4x+2)=7/9 (given that when both numbers are increased by 2 the ratio
becomes 7/9)
=> 9(3x + 2 ) = 7(4x + 2)

=>

-2x = -20

=> 27x + 18 = 28x + 14

=>

x =10

=> 27x - 28x = 14 -18

Therefore, the student in each class before the increase are 2x10, 3x10, 5x10 = 20,30,50

=>

And total student = 100

=>

- x

= -4

x =4

Thus, the first number is 4 x 3 =12

10. The ratio between two number is 3:4 ,if each number be increased by 9, the ratio
becomes 18:23 find the sum of the number

and the seconder number is 4 x 4 =16


9. The students in three classes are in the ratio 2:3:5. If 20 students are increased in
each class the ratio changes to 4:5:7. what was the total number of students in the three
classes before the increases?
(a) 120

(b) 85

(c) 72

(d) 100

(a) 135

(b) 105

(c) 155

(d) 165

Answer: (b) 105


Solution: Let the two numbers be 3x and 4x.

Answer: (d) 100

When they are increased by 9 they become 3x + 9 and 4x + 9.

Ctethelper.blogspot.com

Ctethelper.blogspot.com

It is given that the ratio is 18:23

X=5

Thus, 3x + 9/4x + 9 = 18:23

Thus, 5 should be added.

23(3x + 9)

= 18(4x + 9)

69x + 207

= 72x + 162

12. Find the number which, when subtracted from the terms of ratio 11:23 makes it
equal to the ratio 3:7

69x 72x = 162- 207

(a) 2

(b) 8

-3x = -45

(c) 16

(d) 6

X = 15
Thus two numbers are 3x15 = 45 and 4 x 15 = 60
And the sum is 60+45 = 105

Answer: (a) 2
Solution:

11. Find the number which, when added to the terms of the ratio 11:23 makes it equal to
the ratio 4:7
(a) 15

(b) 5

(c) 25

(d) 20

Let the number be x which when subtracted from each terms of the ratio 11:23 makes it
equal to 3:7.
Thus, (11 - x)/(23 - x ) = 3/7

Answer: (b) 5

7(11 - x ) = 3(23 - x)

Solution:

77 - 7x = 69 - 3x
-7x+3x = 69 77

Let the number be x which when added to each terms of the ratio 11:23 makes it equal to
4:7.

-4x = -8
X=2

Thus, (11 + x)/(23 + x ) = 4/7


Thus, 2 should be subtracted
7(11 + x ) = 4(23 + x)
77 + 7x = 92 + 4x

13. In 40 liters mixture of milk and water the ratio of milk and water is 3: 1 how much
water should be added in the mixture so that the ratio of milk to water becomes 2:1 ?

7x-4x = 92 77
(a) 500ml

(b) 20 liters

(c) 5 liters

(d) 10 liters

3x = 15

Ctethelper.blogspot.com

Ctethelper.blogspot.com
x=4

Answer:

Therefore, the milk in the mixture is 4x3 = 12 litres and quantity of water = 4x2 = 8
liters

Solution:
15. A bag contain equal number of one rupee, 50 paisa and 25 paisa coins respectively if
the total value of rs 35, how many coins of each type are three?
Let x liters of water be added to the mixture to make the ratio of milk to water to become
2:1.

(a) 40

(b)80

(c) 160

(d) 20

Thus,
30/(10 + x) = 2/1
Answer: (d) 20
30(1) = 2(10+x )
Solution:
30 = 20 + 2x
10 = 2x
50 paisa = .5 rupee. .25 paisa = .25 rupees.
X = 5 liters
Now, given that
14. A mixture contains milk and water in the ratio of 3:2 liter of water is added to the
mixture, milk, milk and water in the mixture become equal find the quantities of milk
and water in the mixture .
(a) 12, 8 liter

(b) 4,3 liter

(c) 12, 6 liters

(d) 10,8 liters

X (1 + 0.50 + 0.25) = 35
=> x = 35/(1.75) = 20

Thus, there are 20 coins of each type.

16. The speed of these cars is in the ratio of 2:3:4. what is the ratio among the time taken
by these cars to travel the same distance ?

Answer: (a) 12, 8 liter


(a) 7:8:12

(b) 3:6:9

(c) 9:3:6

(d) 6:4:3

Solution:
Let quantities of milk and water in the mixture be 3x and 2x. Then if 4 liters of water is
added to the mixture the ratio of milk and water become 1:1.
It can be written as (3x): (2x + 4) = 1/1

Answer: (d) 6:4:3

Thus, 3x = 2x +4

Solution:

Ctethelper.blogspot.com

Ctethelper.blogspot.com

Let the distance be 2x3x4 = 24 km.

(a) 11:22

(b) 71:58

Then, ratio of time taken by each car is 24/2: 24/3: 24/4

(c) 31:74

(d) 91:49

12:8:6
or 6:4:3

Answer:(c) 31:74
Solution:

17. The incomes of A of B are in the ratio 3:2 and their expenditure are in the ratio 5:3 if
each saves rs 2000, what is their income ?

In vessel 1
The fraction of water = 1/3

(a) 32000

(b) 20000
The fraction of milk = 2/3

(c) 1190

(d) 8000
In vessel 2
The fraction of water = 2/7
The fraction of milk = 5/7

Answer: (d) 8000


Solution:
Let the income be 3x and 2x. It is given that the saving of each is Rs. 2000.
Then, their expenditures are 3x 2000 and 2x 2000

Now, From vessel 1, 1/5 of the mixture is taken out. Thus water taken out = (1/5)(1/3)
= 1/15 and milk taken out = (1/5)(2/3) = 2/15
And from vessel 2, 4/5 of the mixture is taken out. Thus water taken out = (4/5)(2/7) =
8/35 and milk taken out = (4/5)(5/7) = 20/35

Again, (3x 2000)/(2x 2000) = 5/3


=> 3(3x 2000) = 5(2x 2000)
=> 9x 6000 = 10x 10000
=> 9x -10x

Thus Ratio of water and milk in resulting solution is (1/15 + 8/35) : (2/15+20/35) =
31/105 : 74/105
= 31:74

= -10000+ 6000

=>

-x = -4000

=>

x = 4000

19. Find the mean proportion of 9, 16


(a)12

(b) 144

(c) 25

(d) 4/3

Therefore, their salaries are 3 x 4000 = 12000 and 2 x 4000 = 8000


18. The contents of two vessels containing water and milk are in the ratio 1:2 and 2:5
are mixed in the ratio 1:4, the resulting mixture will have water and milk in the ratio.

Ctethelper.blogspot.com

Ctethelper.blogspot.com

Answer:(a)12
22. The ratio of three number is 3:4:5 and the sum of their squares is 1250. The sum of
their number is :

Solution:
Mean proportion of 9 and 16 = (9 x 16) = 3 x 4 = 12
20. Find the third proportional to 15 and 20
(a)80/3

(b) 55/6

(c) 37/2

(d) 71/2

(a) 30

(b) 50

(c) 60

(d) 90

Answer: (c) 60
Solution:Let the numbers be 3x, 4x and 5x.

Answer:(a)80/3

Then,

Solution:

Given that sum of their squares is 1250

Here, given one ratio

=> (3x) + (4x ) + (5x) = 1250

15:20 and 20:x . We have to find the value of x.

=> 9x + 16 x + 25x =1250

Now 15/20 = 20/x

=> 50x = 1250

=> x = (20x20)/ 15 = 400/15 = 80/3

=> x = 25
=> x = 5

21. Find the fourth proportional to the number 6,8 and 15


Thus, the number are 3 x 5 = 15, 4 x 5 = 20 and 5 x 5 = 25
(a) 48

(b) 48

(c) 41

(d) 20

Ad therefore their sum = 60

23. The ratio of three number is 3:4:7 and product 18144 the number are :
Solution:
(a) 9,12,21

(b) 15,20,25

(c) 18,24,42

(d) None of the

Let the fourth proportional be F.


Then,
6/8 = 15/F
Answer:(c) 18,24,42
=> F = (15 x 8 )/6 = 120/6 = 20
Solution: Let numbers are 3x, 4x and 7x

Ctethelper.blogspot.com

Ctethelper.blogspot.com

Given that the product of the numbers is 18144


Therefore, (3x)(4x)(7x) = 18144

25. What should be subtracted from 15,28,20 and 38 so that the remaining number may
by proportional?

=> 84x = 18144

(a) 2

(b) 4

=> x = 216

(c) 6

(d) None of the

=>

x=6

Therefore the numbers are : 3 x 6 = 18


4 x 6 = 24
and

7 x 6 = 42

Answer: (a) 2
solution:
Let k be subtracted from 15,28,20,38 such that yhey may be proportional. then
(15 - k)/(28-k) = (20 - k)/(38 - k)

24. The value of K that must be added to 7,16,43,79 so that they are in proportion is.
=> (15 - k)( 38-k) = (20-k)(28-k)
(a) 7

(b) 5

(c) 9

(d) None of the

=> 570 - 53k + k = 560 - 48k +k


=> 570 - 53k = 560 - 48k
=> 570- 560 = 53k - 48k
Answer: (b) 5
=> 10 = 5k
Solution:
=> k = 2
k is added to 7,16,43,79 to make them in proportion then
(7 + k)/(16 + k) = (43 + k)/(79 + k)

26. Two number are respectively 20% and 50% more than a third number. The ratio of
two numbers is :

=> (7 + k)(79 + k) = (43 + k)(16 + k)


(a) 2:5

(b) 3:5

(c) 4:5

(d) 6:7

=> 553 + 7k +79k + k = 688 + 43k + 16k + k


=> 553 + 86k = 688 + 59k
=> 553 - 688 = -86k + 59k
Answer:(c) 4:5
=> -135 = -27k
Solution:
=> k = 5
Let the third number be x.

Ctethelper.blogspot.com

Ctethelper.blogspot.com

Then the first number = (1.2) x

Solution:

And the second number = (1.5)x

Share of C = (4/9)x 900 = 4 x 100 = 400

Therefore their ration = (1.2x)/(1.5x) = 12/15=4/5 or 4 : 5


29. If x:y is 9:7 then (x+y):(x-y) is ......
27. A and B together have Rs 1210, if 4/15 of A's amount is equal to (2/5) of B's amount.
How much amount does B have ?
(a) Rs.460

(b) Rs.484

(c) Rs.550

(d) Rs.664

(a)8:1

(b) 1:8

(c)4:1

(d) 1:4

Answer:(a)8:1
Solution:
Answer: (b) Rs.484
(x+y) :(x-y) = {(x/y) + 1 } /{(x/y) -1} = {9/7 +1}/{9/7 - 1} = 16/2 = 8/1 or 8:1
Solution:
Let A's amount to be a.
30. The ratio of the present ages of Sunita and Vinita is 4:5. Six years hence, the ratio of
their ages will be 14:17. What will be their ages 12 years hence.

And B's amount be b.


Given that (4/15)a = (2/5)b => a = (2/5)(15/4)b = (3/2)b
Thus, a + b = 1210

(a) 13:19

(b)16:19

(c) 17:19

(d) 15:19

=> (3/2)b + b = 1210


=> (5/2)b = 1210

Answers: (b)16:19

=> b = (1210)(2/5) = (242)x 2 = 484

Solution:
The word to understand here is 'hence'. 'Hence' in such questions means 'after' the years
mentioned.

28. Rs. 900 is to be distributed among A,B,C and in the proportion 2:3:4 how much C
get?
(a) Rs.400

(b) Rs.450

(c) Rs.540

(d) Rs.None of the

Let the present ages of Sunita and Vinita be 4x and 5x. Now, according to the question,
(4x + 6)/(5x + 6 ) = 14/17
=> 68x + 102 = 70x + 84
=> 102-84 = 70x - 68x

Answer:(a) Rs.400

=> 18 = 2x

Ctethelper.blogspot.com

Ctethelper.blogspot.com

=> x = 9
Therefore, Sunita's age = 4x9 = 36 years
Vinita's age = 5x9 = 45
After, 12 years Sunita's age = 36+ 12 = 48 years
12 years Vinita's age = 45+ 12 = 57 years

32. From each of the two given unequal numbers, half the smaller number is subtracted.
Then, of the resulting numbers, the larger one is five times than the smaller one. Then
the ratio of the larger to smaller one is
(a) 2 : 1
(b) 3 : 2
(c) 3 : 1
(d) 1 : 4

And ratio of their ages = 48/57 = 16/19 or 16:19


Answer:(c) 3 : 1
31. The ratio of the two numbers is 3:4 and their LCM is 180. The second number is
Solution:
(a) 45

(b) 90

(c) 30

(d) 60

Let the numbers be X and Y and X >Y.


Then according to the conditions given in the problem, we have
(X-Y/2)/(Y/2) = 5/1

Answer: (d) 60
Solution:

=> (2X-Y/(Y)) = 5/1

The point to understand here is that,

=>

2X-Y = 5Y

when we find ratio of two numbers then the HCF of the numbers is eliminated while
reducing the ratio to the lowest term. Thus, the two numbers are 3 x HCF and 4 x HCF

=>

2X = 6Y

=>

X/Y = 6/2 = 3/1

Now, Let the HCF be x. Then using the formula


or X:Y = 3:1
LCM x HCF = First Number and Second Number
180 x HCF = (3xHCF)(4xHCF)
Click here for more problems on
=> 180/(3x4) = HCF
=>

15 = HCF

Therefore, the second number is 4 x15 = 60

Time and Work


Time Speed and distance

Ctethelper.blogspot.com

Ctethelper.blogspot.com

UNITARY METHOD
Unitary method is about calculating for a one item and then using it to get the solution
for the given number of quantity.
2. If 4 men or 7 women do a work in 60 days then in how many days will 8 men and 7
women finish the same work
For example:

(a) 18

(b) 20

(c) 48

(d) 36

If the cost of 30 m of cloth is 345, then find the cost of 16 m of such cloth?
Solution:

Answer:(b)

Given that the cost of 30 m of cloth is: Rs 345

Solution :

Then the cost of 1 m of cloth is (345/30) = 11.5

In this problem, we to calculate the work done by a single men or a single women in one
day and then we can easily find the answer to the above problem. We assume that the
total work done is 1 unit.

Therefore, the cost of 16 m of cloth is (11.5) x 16 = 184

Work done by 4 men in one day = 1/60

SOLVED PROBLEMS
1. If the cost of 30 meters of cloth is Rs 345 . Find the cost of 16 meters of this cloth.
(a) 184

(b) 162

(c) 875

(d) 321

Work done by a single men

= 1/(60 x 4) = 1/240

Work Done by 7 women in 1 day = 1/60


Work Done by 7 women in 1 day = 1/(60 x 7) = 1/(420)
Now, from the condition given in the problem,

Answer:(a)

Work done by 8 men and 7 women in a single day is 8 x (1/240) + 7 x(1/420) = 1/30 +
1/60 = 3/60=1/20

Solution:
Cost of 30 m of cloth = Rs 345
Cost of 1 m of cloth = Rs 11.5
Cost of 16 meter of cloth = 16 x 11.5 = 184

Time taken to do the work = 1/1/20 = 20 days.

Ctethelper.blogspot.com

Ctethelper.blogspot.com
Solution:

3. A contractor undertook to do a work in 60 days employed 50 workers to carry the job


but after 40 days he found that only half had work had been done now how many more
worker should he employ to finish the work in time ?

6 people spent 2400 in 4 month


6 people will spent in 1 month = 2400/4 = 600

(a) 81

(b) 50

(c) 20

(d) 26

1 person spent in 1 month = 600/6 = 100

Now to find how many month it will take to 4 person to spent 1800.
Since spent of 1 person is 100 and 4 person spent is 100 x 4 = 400
Answer: (b)
And the month it will take to spent 1800 is = 1600/400 = 4 months
Solution:
The hey here is only half the work is done in 40 days by 50 workers.
Now we will find the work done by a 50 workers in 40 days = (1/2)

5. If 20 men can complete a work in 3 days while 12 ladies can complete the same work
in 12 days .in how many days will 20 men and ladies complete the same work ?

And lets say x workers can do the remaining half of the work in 20 days.

(a) 2/17

the equation thus becomes

(c) 36/17

(b) 2/5
(d) 17/3

X x 20 = 50 x 40
X = 100

Answer:(c)
Solution:

And since 50 workers are already employed, The contractor has to rope in 100 - 50 = 50
more workers to do the job.

First find the work done by 1 men and 1 lady in 1 day


Lets assume work done is 1 unit.

4. If 6 people spend rs 2400 in 4 months then 2 less people will spend Rs 1600 in how
many month?

Now,
Work done by 1 men in 1 day = 1/(20 x 3) = 1/60

(a) 4

(b) 6

(c) 63

(d) 24

and work done by 1 lady in 1 day = 1/(12 x 12 ) = 1/144


Work done by 1 men and 1 lady in 1 day = 1/60 + 1/144 = 17/720
Answer:(a)

And Work done by 20 men and 20 lady in 1 days = (17/720)x 20 = 17/36

Ctethelper.blogspot.com

Ctethelper.blogspot.com

And days it will take to complete the work = 1/(17/36) = 36/17

Solution:
Rate of exchange for 1 dollar = 7.5/100
And for 550 we will get = (7.5/100) x 550 = 41.25 dollars

6. If each boy takes two chocolates a day it will suffice to 7 boys for 12 days if each boy
takes three chocolates a day then for how many days will suffice to 8 boys ?
8. If the weight of 13 meters long rod is 23.4 kg what is the weight of 6 meter long rod ?
(a) 10

(b) 15

(c) 16

(d) 7

(a) 7.1 kg

(b) 10.8 kg

(c) 12.4 kg

(d)18.0 kg

Answer: (d)
Answer: (b)
Solution:
Solution:
First calculate the total chocolate availabe.
First Calculate the weight of 1 m rod = 23.4/13 = 1.8 kg/m
There are 7 boys and they take 2 chocolate each day for 12 days = 7 x 12 x 2 = 168
And the weight of 6 m rod is = 1.8 x 6 = 10.8 Kg

Now there are 8 boys now and they each take 3 chocolate each day there total chocolate
eaten in day = 24

10. If a scale of map represents 0.8 cm for 8.8 km if two points in a map are 80.5 cm
apart what the actual distance between these points ?
(a) 885.5 km

(b) 905 km

(c) 700 km

(d) 990 km

and they will last 168/24 = 7 days


7. If the rate of exchange is 7.50 dollars for Rs 100 how many dollars be equal to Rs 550
?
Answer:(a)
(a) 41.75

(b) 42.75

(c) 41.25

(d) 42.25

Solution::
First calculate how much 1 cm represents = 8.8/0.8 Km
Therefore 80.5 cm represents = (8.8/0.8) x 80.5 = 885.5 Km
Answer: (c)

Ctethelper.blogspot.com

Ctethelper.blogspot.com

11. If 45 men can complete work in 30 days working 12 hours a day in how many day
work 60 men complete the work working 10 hours days ?

There fore the provision will last 42 days more after 25 men left.

(a) 31

(b) 27

(c) 25

(d) 33

Answer: (b)

13. If 6 men weave 168 shawls in 7 days how many shawls will be woven by 8 men is 5
days ?
(a) 126

(b) 90

(c) 160

(d) 104

Solution:
Work done by 1 men per day per hour = 1(/ 30 x 12 x 45)
Lets say it takes X days to complete the work if they work 10 hours a day and there are
60 men

Answer: (c)
Solution:

then, work done by a men per hour comes out to be 1/ ( 10 x 60 x X)


Shawls woven in 1 day by the 6 men = 168/7 = 24
Equating both the equations to get
And Shawls woven in 1 day by 1 men = 24/6 = 4 shawls
30 x 12 x 45 = 10 x 60 x X
= > X = 27 days
Therefore ,
12. A fort provision for 150 men for 45 days after 10 days 25 men left fort how long will
the provision last from that day?

8 men will weave 4 x 8 = 32 shawls in 1 day


and in 5 days they will weave 5 x 32 = 160 shawls.

(a) 45

(b) 54

(c) 29 /15

(d) 42
14. If 51 people can eat some food in 50 days how many days will the same food be eaten
50 people ?

Answer: (d)

(a) 52

(b) 51

Solution:

(c) 50

(d) 55

Total provision in the fort = 150 x 45


After 10 days provision left = 150 x 35 = 5250

Answer: (b)

Now, there are 125 men left and it will last = 5250/125 = 42 days

Solution:

Ctethelper.blogspot.com

Ctethelper.blogspot.com

Since Total food remains the same which is 51 people X 50 men.

And lets says he employs a total of M men to complete the rest of the half work in
30days

Now there are 50 people there days = 51x 50 / 50 = 51 days


Then
M x 30 = 50 X 30
16. If 15 oxen or 20 cows can eat the grass of a field in 80 days then in how many days
will 6 oxen and 2 cows eat the same grass ?

M = 50 men

(a) 40

(b) 60

And men that were added to complete the work in time = 50 30 = 20

(c) 100

(d) 160

Answer:
Solution:
grass eaten by 1 oxen in 1 day = 1/(15 x 80)

18. A family has provision for 15 people for 1 week for if after 2 days 5 people are
increased and the ration is reduced by 25 % per person how long will the provision last ?

grass eaten by 1 cow in 1 day = 1/(20 x 80)


(a) 5

(b) 6

(c) 7

(d) 4

Let it takes x days to eat the grass, then


x { 6/1200 + 2/1600) = x ( 30/4800) = x (1/160) = 1
And therefore x = 160 days
Answer:
Solution:
17. A contractor undertook at do a work in 80 days he employed 30 men to carry out the
job but after 50 days he found that only half work had been done now how many more
men should he employ to finish the work in time ?

Lets ration per day is R then accodring to the conditions given in the above problem, we
have

(a) 30

(b) 50

15 x 7 x R = 15 x R x 2 + 20 x 0.75R x D

(c) 20

(d) 10

105 = 30 + 15 D
D=5

Answer: (c)
Solution:
Half work was done in 50 days employing 30 men

Therefore Ration will last 5 more days after intial 2 days are over. Therefore total days =
7 days

Ctethelper.blogspot.com
19. A regular working day is 8 hours and regular week is 5 working days .A man is paid
rs 2.40 per regular hour and rs 3.20 per hour overtime if he earns rs 432 in 4 week what
is the total number of hours he works ?
(a) 180

(b) 195

(c) 160

(d) 175

Ctethelper.blogspot.com

21. If 3/5 of a work is done by 12 men in 10 days how many men will complete the whole
work in 20 days?
(a) 9

(b) 8

(c) 10

(d) 7

Answer: (d)
Answer:
Solution:
Solution:
Total hours in 4 weeks for a regular week = 8 x 5 x 4 = 160 hours
Total work done would have been done in (5/3) x 12 x 10 days
Amount he should get without overtime = 160 x 2.4 = 384
Let M men could complete the work in 20 days
He got 432, an extra 48 have come due to overtime. Rate of overtime is 3.20
Then,
Therefore overtime is done for 48/3.2 = 15 hours
Equating both the terms
Therefore total hours = 160 + 15 = 175
We have,
(5/3) x 12 x 10 = M x 20
20. If one fourth kg of a substance costs rs 0.80 what will be the cost of 200 gram of the
same substance?
(a) 40 paisa

(b) 100 paisa

(c) 64 paisa

(d) 60 paisa

Answer: (c)

=> M= 10 men

22. 400 persons working 9 hours a day complete 1/4 the of a piece of work in 10 days
the number of additional persons working 8 hours a day required to complete the
remaining work in 20 days is ....(a) 675
(b) 275
(c) 250

(d) 225

Solution:
Cost of 1/4 kg of substance = 0.80

Answer: (b)

Cost of 1 kg of substance = 0.80/.25 = 3.2

Solution:

And therefore cost of 200 g = 3.2 X .2 = 0.64

Work done by 400 people working 9 hours a day for 10 days = 400 x 9 x 10 = 1/4 of total
work

Ctethelper.blogspot.com

Ctethelper.blogspot.com

Let total work be T

Arithmetic Average

Then T= 4 x 400 x 9 x 10
And work remaining = 3/4 T= (3/4) x 4 x 400 x 9x 10
The formula for calculating average is :
Again Let M is the number of total men after the addition of men to complete the work
in 20 days, then
(3/4) x 4 x 400 x 9 x 10 = M X 20 x 8

Average = (x1 + x2 + x3 +x4 +x5 + x6 + ... + xn)/n

108000 = M x 160
M = 675
Therefore Men increased by = 675 - 400 = 275

where,
n is the number of quantities
x1 is the value of first quantity, x2 is the value of second
quantity and so on.

Example: There are seven boxes having weight of 7, 6 , 8, 5, 4 , 6 ,8.


Calculate
the average of all the boxes.

Solution: Using the above formula, we have seven boxes, therefore n =7


and x1 = 7, x2 = 6 and so on
put the values in the above formula to get

average = ( 7+ 6 + 8 + 5 +4 +6 +8)/7 = 44/7 = 6.28 approx.

Weighted Average

Ctethelper.blogspot.com

Ctethelper.blogspot.com

In case of weighted average we assign different weights to each quantity


and then calculate their average.

= 66.67

and an arithmetic average would have been


Formula for weighted average is
= (210)/3= 71

Average (weighted) = (w1x1 + w2x2 + w3x3 + .... + wnxn)/(w1 + w2 +


w3 +.. + wn)
Comparison between Weighted average and arithmetic average:

Where,
w1,w2,w3....wn are the weights of x1,x2,x3,...xn quantities.

The weighted average calculated in this case is less than the arithmetic
average because of the weight attached to the various subjects. Since
Maximum weight is attached to physics, and the student has scored less
mark in physics, this will pull down the average.

Example:
Take another scenario where admission is sought in chemistry course and
the weight attached are as:
A student scored 60, 80 and 70 in Physics, chemistry and math. If the
college assigns a weight of 3 to physics, 1 to chemistry and 2 to math for
admission to the physics course, then calculate the weighted average of the
student.

Solution:

Math

= 1

Physics

= 2

chemistry

=3

Then the weighted average would be = ( 120 + 240 + 70)/6 = 71.67


Here,
Weights of marks in physics, chemistry and math are 30, 40 and 80
respectively.

Now weighted average has come out to be greater than arithmetic average
because of the weight assigned to chemistry and the higher marks in
chemistry has pull UP the weighted average.

Therefore using the formula for the weighted average, we have,

weighted Average = (3x60+ 1x80 + 2x70)/(3 + 1 + 2 )


= (180 + 80 + 140)/(6)

Solved Problems on Averages


Question 1: The average age of 20 students in a class is 12 years. If the age of the
teachers is also added then the average becomes 13 years. Find out the age of the
teacher?

Ctethelper.blogspot.com

Ctethelper.blogspot.com

Solution:

Solution:
Here simply use the average formula :

Method 1: solving by using the formula.


Average = (10 x 250 + 15 x 350 + 20x50)/(45) = (2500 + 5250 + 1000)/45
Let the age of the teacher be X years.
= 194.44
Then the formula for averages becomes
13 = (20 x 12 + X)/21 = (240 + X)/21
Multiplying both the sides by 21 to get

Question 3: The average contribution of an office towards the health fund is 120 month.
If the officer level employees are contributing Rs. 460 and non-officers staff is paying Rs
110. Then, if there are 15 officers then find the number of non-officer staff.

=> 273 = 240 + X


Solution:
on solving for X
X = 33 years
So the age of the teacher is 33 years

Let the non-officer staff be X.


Then total staff would be (15 + X)
Then total contribution by all the staff = 120 (15 + X )

Method 2 :
Using the formula for the average calculation
Adding the teachers age to the group increases the age of the students by 1 full year.
120 (15 + X ) = 6900 + 110 X
Since there are 20 students the total years added by the inclusion of the teacher = 20
years
And since the teacher is new addition to the group having an average of 13, 13 years
more years are added.

Solving for X
1800 + 120X = 6900 + 110X
120X - 110X = 6900 - 1800 = 5100

Therefore total years added to the group due to the inclusion of the teacher = 13 + 20 =
33 years.

10X= 5100
X = 510
There fore non-Officer staff is 510

Question 2: Rakesh bought 10 shirts for 250 each, 15 pants for 350 each and 20
handkerchiefs for 50 each. Find the average cost of each item.

Question 4: A certain college while taking admission in the math course is assigning
following weight age to the subjects.

Ctethelper.blogspot.com

Ctethelper.blogspot.com

Math - 4
Question 5: One-third of the journey is covered at the rate of 25 km/hr , one-fourth at
the rate of 30 km/hr and the rest at 50 km/hr. Find the average speed for the whole
journey.

Science - 3
English - 2
Hindi - 1

Solution:
If a student has secured following marks in these subjects then find out arithmetic
average and the weighted average of the marks.

For calculating speed we have to first find the total distance covered and total time
taken.

Math - 70

Let the total distance be X km.

Science - 60

Then according to the question

English - 70

1. (X/3) distance is covered at 25 Km/hr and time taken = X/75 hr

Hindi - 80

2. (1/4)X distance is covered at 30 Km.hr and time taken = X/120

Solution:

3. Rest of the journey ( X - ( X/3 + X/4)) = 5X/12 at the rate 50 Km/hr and time taken =
X/120

Using the Formula for arithmetic averages we have,

Therefore calculating the average time taken

Arithmetic average = (70 + 60 + 70 + 80) / 4 = 70


Average Speed = Total Distance Covered/Total Time Taken
= X/ ( X/75 + X/120 + x/120)

Using the formula for the weighted average we have

= 1/(3/100) = 100/3

weighted average = ( 4 x 70 + 3x60 + 2x70 + 1x80)/(4 + 3 + 2 + 1)

= 33(1/3) Km/Hr

= (280 + 180 + 140 + 80)/10


= 68
Difference between the arithmetic and the weighted average = 2
and it is due to the different weight age to various subjects in calculating the weighted
average.

Question 6: The average weight of 8 persons is increased by 2 Kg when one of them who
weighs 56 kg is replaced by another person. The weight of the new person is :
Solution:

Ctethelper.blogspot.com

Ctethelper.blogspot.com
Total marks in five subjects = 335

The new person added 2 kg to the average weight of 8 people. Then total new weight
added to the group is 8 X 2 = 16 kg. In addition to this weight , The new person also
replaced a member with 56 kg of weight. Then, then total weight of the new person = 56
+ 16 = 72 Kg.

Therefore average marks in five subjects = (335)/5 = 67

Question 9: The average of 13 results is 50. If the average of the first 6 results is 49 and
that of the last 6 is 53, then find the 7th results.
Question 7: In an examination, one section of class X, having 60 students scored an
average marks of 55 and the second section, having 40 students scored an average of 45
marks.What is the overall average.

Solution:

Solution:

Given that the averages of the first 6 results and the last six result are 49 and 53
respectively.

Average = Total Marks scored by all the students / Total numbers of students

Then the total value of all the 12 result ( except the 7 th one) is = 49x6 + 53x6 = 612
And the total value of all the 13 result = 650

1. Total Marks scored by 60 students of first section = 55x60 = 3300


2. Total marks scored by 40 students of second section = 40 X 45 = 1800

Therefore the seventh value would be = 650 -612 = 38

Therefore total marks scored by all the students = 5100


and total students = 60 + 40 = 100
Using the formula for average mentioned above,

Question 10: A driver drives to the office at 50 km/her and returns at 30km/hr. Find the
average speed.

we have ,
Solution:
average marks = 5100/100 = 51
The key here is that the driver drives the same distance to and fro. Let the distance be D
in one direction.
Question 8: The average marks of a student in four subjects is 65. If the student scored
75 marks in the fifth subject then the average of the total marks is ?

Then the total distance travelled wound be 2D.


Now, time taken while driving at 50 km/hr = D/ 50 hr

Solution:

And time taken to while driving at 30 km/hr = D/30 hr

Marks in the four subjects = 65 x 4 = 260


Now,
Marks in the fifth subject = 75

Ctethelper.blogspot.com

Ctethelper.blogspot.com

Average Speed = Total Distance Traveled/ Total Time Taken = 2D/(D/ 50 + D/30)
Answer: (b) 80 years
= 2/8/150 = 150/4 = 37.50 km/hr
Solution:
Since Addition of D to the group reduces the average age of the group to 80 years. This
means addition of D, reduces 4 x 4 = 16 years from the total age. Thus, the age of D is 84
- 16 = 68 years.
Q 11. Out of 30 teachers of a school, a teacher of age 60 years retired. In his place
another teacher of 30 years was appointed. As a result , the mean age of the teachers will
(a) decrease by 1 year

(b) remain same

Again, given that the age of E is 4 years less than D then the age of E is 68 + 4 = 72
years.

(c) decrease by 2 years (d) decrease by 6 month.


Addition of E and exit of A reduces the group average to 78 years. This means A
total of 4x2 = 8 years have been taken from the group total. Thus, age of A is 72 + 8 = 80
years.

Answer:
Solution:
First Method:
Net result of the retirement and appointment of the teachers result in a net decrease of
30 years. These 30 years are distributed across 30 teachers. Thus, the mean decreases
by 1 years.

Q13. The average of six numbers is 32. If each of the first three numbers is increased by
2 and each of the remaining three numbers is decreased by 4, then the new average is
(a) 35
(b) 34
(c) 31
(d) 30
Answer:(c) 31

Second method:

Solution:

Let the mean of the 29 teachers that remain the same be x years, then

Each of the first three numbers is increased by 2 then the increases in total

Final Mean - Initial mean = (29x +30 )/30 - (29x + 60)/30 = (29x + 30 -29x - 60)/30
= - 30/30 = - 1

= 3 x 2 = +6
And each of the 3 remaining three numbers are decreased by 4, then total decreased is 4
x 3 = - 12

Thus, the final mean is less than the initial mean by 1 year.
Therefore, net increase/decrease = + 6 - 12 = -6
Q12. Average age of A, B and C is 84 years.When D joins them the average age becomes
80 years. A new person E, whose age is 4 years more than D replaces A and the average
of B, C , D and E becomes 78 years. What is the age of A?
(a) 70 years

And the decrease in the mean = -6/6 = -1. And new mean = 32- 1 = 31

(b) 80 years

(c) 50 years (d) 60 years

14. Five years ago, the average age of P and Q was 25. The average age of P, Q and R
today is 25. Age of R after 5 years will be

Ctethelper.blogspot.com
(a) 15
(c) 40

Ctethelper.blogspot.com

(b) 20
(d) 35
Faster Method:
Due to the error in reading, the total reduced by 64-46= 18. And the loss of 18 was
distributed among 36 members. That is average reduced by 19/36 = 0.5. Therefore
correct average is 52+ 0.5 = 52.5

Answer: (b) 20
Solution:
Sum of ages of P and Q, five years ago, is {(P - 5) + (Q - 5 ) = 50 years}
present ages =P + Q = 60 years

And their

Again, When R is added to the group then sum = (P + Q + R) = 60 + R


and average = (60 + R)/3 = 25 (given)
=> 60 + R = 75
=> R =15 years
And age of R after 5 years = 15+ 5 = 20 years

15. The average marks scored by 36 students was 52. But it was discovered that an item
64 misread as 46.What is the correct mean of marks?
(a) 54

(b) 53.5

(c) 53

(d) 52.5

Answer:(d) 52.5
Solution:
First method:
Total marks of the student with the misread item = 36 x 52 = 1872
Total marks of the student with the correct item = 1872 - 46 + 64 = 1890
Therefore, average = 1840/36 = 52.5

Ctethelper.blogspot.com

Ctethelper.blogspot.com

SPEED TIME AMD DISTANCE

(a) 2 min

(b)9 min

(c) 1 min

(d) 7 min

1. A train is running with the speed of 45 km per hour? what is its speed in
meter per second ?
(a) 12.5 m/s

(b) 13 m/s

(c) 16 m/s

(d) 81 m/s

(e) None of the

Answer: (c) 1 min


Solution:

(e)None of the

Answer: (a) 12.5 m/s

The faster train is the one that is running at 36 km/hr.

The difference in speed of the two trains is (36 - 21 ) km/hr = 15 km/hr

Solution:

And the combined length of the train is (100 + 150 ) m = 250 m.

Speed of train = 45 km per hour


Convert km to meters= 45 x 1000 = 45000 m

( because 1 km = 1000 m)

Therefore, the time taken is = (250m )/(15 km/hr) = (250 m)/ (15 x 5/18
m/s)

Convert Hour to seconds=

= 60 seconds

60 x 60 seconds = 3600 seconds (because 1 hour = 60 min, 1 min =


60 seconds)

Therefore 45 km/hr = (45000/3600) meter per second =


second

12.5 meter per

Short Cut:

= 1 minutes.

3. A plat form is 131 meter long if the speed of the train, 67 meter long,
is 45 km per hour, how long will it take to pass the platform ?
(a) 15.84 sec

(b) 59 sec

(c) 439 sec

(d) 36.7 sec

(e) None of the

1 Km/hr = (5/18) m/s


Solution:
Simply Multiply the speed in km/hr by (5/18) to get the answer in m/s.
here, 45 km/hr = (45)(5/18) = 12.5 m/s

2. A train 100 m long is running at the speed of 21 km /hr and another train
150 m long is running at the speed of 36 km /hr in the same direction.How
long will the faster train take to pass the first train ?

Here, the platform is stationary. There its speed = 0


The difference in speed between the two objects = 45 - 0=45 km/hr=45
(5/18) m/s= 12.5 m/s
And the combined length of the platform and the train = (131 + 67) =
198 m

Ctethelper.blogspot.com

Ctethelper.blogspot.com

Thus the time taken by the train to pass the platform = 198m/12.5m/s =
15.84 seconds

And the time taken for the second half is = (X/2)/(24) = X/48 hrs --------(ii)

But given that total time taken is 10 hrs. Equating (i) and (ii) to get
4. Find the distance covered by a man walking for 10 minutes at a speed of
6 km/hr.

X/42 + X/48 = 10

(a) 3 km

(b) 4 km

=> X(8 + 7)/336 = 10

(c) 8 km

(d) 1 km

=> X(15) = 3360


=>

X = (3360)/15

Answer: (d) 1 km
=>

X = 224 km/hr

Solution:
Total distance covered = speed x time = (6 km/hr)x(10minutes) =
(6km/hr)(10/60)hr

6. Walking at (3/4)th of his usual speed a person is 10 min late to office.


Find his usual time to cover the distance.
(a) 35 min

(b) 30 min

= 1 Km
(c) 38 min
5. A motor car does a journey in 10 hrs for the first half at 21 km/hr and the
second half at 24 km/ hr. Find the distance?
(a) 244 km

(b) 238 km

(c) 268 km

(d)224 km

Answer: (d)224 km

Answer:

(d) 25 min

(b) 30 min

Solution:
Let the distance between his house and office is X km and his usual speed
be S.
Then, his usual time to reach office = X/S

Solution:
Total distance covered = speed x time

And if he is walking at (3/4)th of his usual speed then the time taken is=
(X)/ (3S/4) = 4X/3S

Let the Total distance be X.

Again, It is given that he was 10 min or 1/6 hours late.

Then, for the first half (X/2) speed was 21 Km/hr.

Therefore the equation thus forms is

Therefore the time taken for the first half = (X/2)/(21 Km/hr) = X/42 hrs -------- (i)

4X/3S - X/S = 1/6

Ctethelper.blogspot.com
=>

Ctethelper.blogspot.com

X/S( 4/3 - 1) = 1/6


Answer: (b) 1.5 km

=> X/S (1/3)= 1/6


Solution:
=> X/S = (1/6)(3/1) = 3/6 = (1/2) km/hr
Distance Covered by A in 3 Hours = (3)(3.5) = 10.5 km
Therefore, the time taken walking at 1/2 km/hr = 2X hours

Distance Covered by B in 3 Hours = (3)(3) = 9 km

And time taken walking at (3/4)(1/2) or 3/8 km/hr = 8X/3 hours.

Thus they will be 10.5 - 9 = 1.5 km apart walking in the same direction.

Again it is given walking at (3/4) speed he is late by 10 min or 1/6 hours


=>

(8X/3) - 2X = 1/6

=> X(2/3) =1/6

8. Two men A and B walk from P to Q , a distance of 21 km, at 3 and 4 km


an hour respectively B reaches Q, returns immediately and meets A at R.
Find the distance from P to R .

=> X = (1/6)(3/2) = 1/4 km

(a) 18 km

(b) 34 km

And usual time = (1/4)/(1/2) = 1/2 hour or 30 minutes

(c) 20 km

(d) 22 km

Short cut:
Answer: (a) 18 km
Given that he is walking at 3/4 of his usual speed.
And Since time taken is inversely proportional to the speed then his time
taken would be (4/3) of his usual time.

Solution:

Then, Let the time be T,

Time Taken by B to reach Q is 21/4 hours.


then

(4T/3) - T = 1/6
(1/3)T = 1/6

In this time, A covered distance of = (21/4)(3) = 63/4 km.


He is still short of 21 - 63/4 = (84-63)/4 = 21/4 km.

T = 3/6 = 1/2 hours or 30 minutes


Now, after reaching Point Q, B turns back to go to point P.
The relative speed of A and B is = 3 + 4 = 7 km
7. Two men A and B start from a place P walking at 3 km and 3.5 km an
hour respectively. How many km will they be apart at the end of 3 hours if
they walk in same direction ?

And distance Between them = 21/4 km.

(a) 6.5 km

(b) 1.5 km

Therefore, time taken to cover this distance = (21/4)/7 = 3/4 hours.

(c) 7.7 km

(d) 3.2 km

Ctethelper.blogspot.com

Ctethelper.blogspot.com

And distance traveled by A in this time to reach point R is = (3/4)3 = 9/4


km.

Let the total distance be X km.

Thus, the distance between Point P and R = (63/4) + (9/4) = 72/4 = 18


km.

Then Peter covered (2/3)X of the distance at 4 km/hr in time = (2X/3)/4


= X/6 hours
And Peter covered (1/3)X of the distance at 3 km/hr in time = (X/3)/5 = (
X/15) hours

9. A man covers a certain distance by car driving at 70 km/hr and he


returns back to the starting point riding on a scooter at 55 km/hr. Find his
average speed for the whole journey.
(a) 31.5 km/hr

(b) 71.5 km/hr

(c) 11.5 km/hr

(d) 61.5 km/hr

Given that total time taken was 1 hour 24 minutes = 1 + 24/60 = 1 + 2/5 =
7/5 hours

There the equation thus formed is

X/6 + (X/15) = 7/5


X(5 + 2 )/30=7/5
X(7/30) = 7/5

Answer: (d) 61.5 km/hr

X=

Solution:
(7/5)(30/7)
For such questions, where the distance covered is same but at different
speeds the formula for the average speed is

X = 6 km

average speed = (2xy)/(x + y), where x and y


are the different speeds.
=> average speed = (2 x 70 x 55)/(70 + 55)
= 7700/125

11. An hero plane flies along the four sides of a square at the speed of 200,
400, 600 and 800 km/hr . Find the average speed of the plane around the
field .
(a) 384 km

(b) 333 km

(c) 324 km

(d) 312 km

= 61.5 km/hr
Answer:(a) 384 km
10. Peter can cover a certain distance in 1 hour 24 minutes by covering two
third of the distance at 4 km/hr and rest at 5 km/hr. Find the total distance.
(a) 6 km

(b) 19 km

(c) 22 km

(d) 32 km

Solution:
For such questions , where the distance covered is the same for all the
speeds, the formula for 4 different speeds is
4abcd/(abc + bcd + cda + dab)

Answer: (a) 6 km
where,
Solution:

same distance

a,b,c,d are the different speeds to cover the

Ctethelper.blogspot.com

Ctethelper.blogspot.com

Therefore, average speed =


{4(200)(400)(600)(800)}/{(200)(400)(600) + (400)(600)(800) +
(600)(800)(200) +
(800)(200)(400)}

(a) 22 min
(c) 26 min

Answer: (d) 19 min

= (400)(2)(4)(6)(8)/{(2)(4)(6) + (4)(6)(8) + (6)(8)(2)+(8)(2)(4)}

Solution:

= (153600)/( 48 + 192 + 96 + 64)

Given that

(153600)/(240 + 160) = 384 km

riding time + walking time = 37 minutes ----------(i)


and

12. A man is walking at a speed of 12 km per hour. After every km he takes


rest for 12 minutes. How much time will he take to cover a distance of 36
km ?
(b) 10 hours

(c) 25 hours

(d) 40 hours

walking time + walking time = 55 minutes

=> 2 x walking time = 55


=>

(a) 30 hours

(b) 28 min
(d) 19 min

walking time = 55/2 = 27.5 minutes

Put this in the equation (i)


Therefore, 27.5 minutes + riding time = 37 minutes
Answer:

riding time = 37 - 27.5 = 9.5 minutes

(b) 10 hours

Solution:
Therefore, total time taken to ride back = 2 x 9.5 = 19 minutes
Total time taken to cover the distance of 36 km (without the rest time) =
36/12 = 3 hours
Now, number of rest he will take to complete the journey = 36 - 1 = 35. 35
because he would have covered the entire distance by the time he takes
36th rest break.

14. A man travels a distance of 18 km from his house to an exhibition by


tonga at 15 km/hr and returns back on cycle at 10 km/hr . Then the average
speed for the whole journey is
(a) 12 km/hr

(b) 10 km/hr

(c) 15 km/hr

(d) 18 km/hr

Therefore total rest taken = 35 x 12 = 420 min = 7 hours.

Thus Total time taken = 3 + 7 = 10 hours.


Answer:
13. I walk a certain distance, ride back taking a total time of 37 minutes . I
could walk both ways in 55 minutes. How long would it take me to ride both
ways .

Solution:
Since the distance traveled is the same for both the journey, therefore using
the formula mentioned in the Short Cut formula for time speed and distance
. we have

Ctethelper.blogspot.com

Ctethelper.blogspot.com
= (30) x {(12)/60}

Average Speed = 2(15)(10)/(15+10) = 2(15)(10)/(25) =


= 6 km

12 km/hr

15. A gun is fired at a distance of 1.34 km from Geeta. She hears the sound
after 4 secs. The speed at which the sound travels is
(a) 330 m/s

(b) 300 m/s

17. Two trains starts from the stations A and B and travel towards each
other at speeds of 50 kmph and 60 kmph respectively. At the time of their
meeting, the second train has traveled 120 km more than the first.
The distance between A and B is

(c) 325 m/s (d) 335 m/s


(a) 1200 km
(c)

Answer: (d) 335 m/s

(b) 1440 km

1320 km

(f)

990 km

Solution:
Answer:(c)

1320 km

Speed of sound = (1.34 x1000 m)/ (4 sec)


Solution:
= (1340 m)/ 4 sec
= 335 m/s

The trains were traveling towards each other then the relative speed is
(50+60)=110 kmph
Again it is given that the second train traveled 120 km more than first.

16. If I walk at 5 km/hr , I miss a train by 7 minutes. However, if I walk at


6 km/hr I reach

The difference in their speed = 60 - 50 = 10 kmph


And time taken to cover 120 kmph = 120/10 = 12 hours.

the station 5 minutes before the departure of the train. The distance
between my house and the station is
(a)

7 km

(b)

6 km

(c)

5 km

(d)

6.5 km

Thus, it took 12 hours for the trains to cover as distance of 110x12 = 1320
km

5. Tom is chasing Jerry. In the same interval of time Tom jumps 8 times
while Jerry jumps 6 times. But the distance covered by Tom in 7 jumps is
equal to the distance covered by Jerry in 5 jumps. The ratio of speed of Tom
and Jerry is

Answer:
Solution:

Using the formula mentioned in the Short Cut formula for time speed and
distance, the required distance is
= {(5x6)/(6-5)} x {(7+5)/60}

(a) 48:35

(b) 28:15

(c) 24:20

(d) 20:21

Ctethelper.blogspot.com

Ctethelper.blogspot.com

Answer: (b) 28:15

TRIANGLES

Solution:
Distance covered by Tom in 7 jump = Distance covered by Jerry in 5 jumps.

Properties of Triangles:

Distance covered by Tom in 1 jump = (5/7) jumps of Jerry.


Therefore, ratio of speed of Tom to Jerry = (8x1)/{6x(5/7)}

1. A triangle has three sides and three interior angles

= 56/30 = 28/15
2. Sum of any two sides of a triangle is always greater than the third side.

i.e. for a triangle ABC, we have

AB + BC > AC
AB + AC > BC
AC + BC > AB

3. Sum of all angles of a triangle is 180.

i.e For a triangle ABC

A + B + C = 180.

Ctethelper.blogspot.com

Ctethelper.blogspot.com
(i) The angles opposite to equal sides are also equal.

4. The exterior angle of a triangle is equal to the sum of the vertically


opposite interior angles.

5. If an angle of a triangle is greater than 90 then the other two angles of


triangle must be acute angles.

3. Equilateral: Equilateral triangle has all its sides equal to each other
Types of triangle

(i) All angles are equal and are of 60 each.

1. Scalene : Scalene triangle is a triangle that has length of its three sides
different from one other.
(i). All angles are different. None of angles are equal.

4. Right Angled: One of the angle of the triangle


is equal to 90.

(i). The side opposite to the right angle is greatest side in the
triangle.
2. Isosceles: Isosceles triangle is a triangle that has two of it's sides equal to
each other.

Ctethelper.blogspot.com

Ctethelper.blogspot.com
the opposite side and perpendicular to it , is called as a height
or altitude. The point where all the altitudes meet is called as
circumcentre.

Terms related to Triangles:

1. Median and centroid : Median is a line that joins a vertex of a triangle to


the mid-point of the opposite side. Median divides the
triangle into two equal halves.If we draw all the medians of a
triangle then they will intersect at a point. That point is called
a centroid.

3. Perpendicular bisector and circumcentre : If we draw perpendicular


bisector of each side, then they will meet at a point known as
circumcentre of the triangle. Circumcentre because if we put
compass on the circumcentre and open the mouth to match the
distance between the circumcentre and any vertex, then we can
be able to draw a circle that contains all the vertices of the
triangle.

2. Height and orthocentre: A straight line from any vertex of the triangle to

Ctethelper.blogspot.com

Ctethelper.blogspot.com
_______________
Area of triangle = s(s-a)(s-b)(s-c)

where,

s = (a +b + c)/2

and a , b, c are the length of the sides of the


triangle.

3. For equilateral Triangle


______
Area = (3 x a )
Formula for calculating area of triangles:

4
where, a is the side of the equilateral triangle

1. Using base and the height:

4. For isosceles Triangle


______

Area of triangle =

xbxh

where, b is the length of base or the side of the triangle where the
altitude from the opposite vertex was drawn
h is the height of the altitude from the vertex to the base.

2. Using Hero's formula

Area = b 4a - b
4
where, a is the length of the equal sides
and b is the length of the third side.

Ctethelper.blogspot.com

Ctethelper.blogspot.com

Solved Questions:
2. In the triangle below angle PQR and angle QRP have the measure of 4m , QSP

1. The perimeter of an equilateral triangle is 723 cm. Find its height.

have the measure of m and RPS has a measure of 45, then what is the measure

(1) 63 m
(2) 24 m
of QPS?

(3) 18 m
(4) 36 m

(1) 45
Solution: Let Perimeter of equilateral triangle = 3 a = 72 3
=> a = 723/ 3 => a = 243

(2) 60
(3) 90

we also know,
Height of equilateral triangle = a (3 / 2)
= 24 3(3/2)
= (24 x 3) /2
= 12 x 3
= 36

(4) 105
(5) 120

Ctethelper.blogspot.com

Solution: QRP is an exterior angle to triangle PRS.

Ctethelper.blogspot.com

and PQS = 4 x 15 = 60

Therefore, QRP = RPS + RSP


Now in triangle PQS, We know that sum of all angles of a triangle =180

=>QRP = 45

+m

Therefore,

QPS + PQS + PSR = 180

But QRP = 4m (given)


=> QPS = 180 - 15 - 60
=> 4m = 45 + m

=> 3m = 45
=> m = 15

=> QPS = 105

Thus,
RSP = 15

3 : In the triangular shown below, if x = 6, then what is the area of the


triangle?
_
(1) 3/8
_

(2) 3/8

Ctethelper.blogspot.com
(3) 3 3/8

Ctethelper.blogspot.com

(4) 3/4

=> (6) = {(6 x 3)/2} + y

(5) 3 3/4

=> (6) = {(18)/2} + y


=>

= {(32)/2} + y

=>

= (3/2) + y

=>

= 9/2 + y

=>

y = 6 - 4.5

=> y = 1.5 = 3/2


=> y

= (3/2)

Solution:
Area of triangle =

xbxh

Putting in the values

Given,
x = 6

Area of triangle = x (3/2) x (3/ 2)


To find,

Area of the given triangle

(3 3/2)

_
= 3 3/4

Since the triangle is a right angled triangle, then using the Pythagoras theorem, we have
i.e. (x) = (x 3/2)

+ y

where 'y' is the base of the triangle in the figure.

Ctethelper.blogspot.com

Ctethelper.blogspot.com

Volume

Cuboid

Volume is the total space that is enclosed by the 3D figures like cuboid, cube, cone and
so on. It is an indication of the space it will occupy or liquid or gases it can store.
A Cuboid has 6 faces just like cube but they are not all equal. For a cuboid at least one of
the length, breadth or height is different .

cube

Cube

Cuboid with length l , height h, and breadth b


Volume of cuboid = l x h x b

A Cube has 6 equal square faces. A good example is a box with equal height, length and
breadth.

Where, l is the length,


h is the height,

Volume of cube = l x l x l ,

and b is the breadth of the cuboid

where l is the length of any side.


Total surface area = 2 ( l x h + l x b + b x h)
Total surface area of cube = 6 X l

Ctethelper.blogspot.com

Ctethelper.blogspot.com
h is the height of cone,

Where, l is the length,


r is radius of circular base
h is the height,
and b is the breadth of the cuboid

Lateral Surface Area =

Total surface area =

Where,

xrxl

x r x l + x r

is approximately equal to (22/7) or 3.14

r is radius of circular base


l is the slant height of cone

Cone

Cone
A cone has circular base and then it tapers with increasing height to a point.
A good example is ice cream cone, party hats.

Volume of cone = (1/3) x

x r x h
Cylinder: A cylinder has circular base and then it extends to a certain

Where, is approximately equal to (22/7) or 3.14

height "h". Good example is soda can, cookie jars, sanitation pipes.

Ctethelper.blogspot.com

Volume of cylinder:

Ctethelper.blogspot.com

x r x h

Where,

= 3.14 or 22/7
r is the radius of the
circular base

Sphere

h is the height of the cylinder

A good example is cricket ball or tennis ball

Lateral Surface area is the surface area of the side( in figure it is the area in yellow
color). It does not include the area of the base and the top
(area in the blue color).

Volume of sphere = (4/3)

Lateral Surface area = 2 x

xrxh

Total surface area = Lateral surface area

x r

Where,

area of the circular base and top

is approximately equal to

(22/7) or 3.14
Total surface area = 2 x

x r x h + 2 x x r

r is radius of sphere

= 2 x x r ( h + r)
Total surface area =
where,

is approximately equal to (22/7) or 3.14

Sphere
A sphere has round 3D figure.

r is radius of circular base

Ctethelper.blogspot.com

Ctethelper.blogspot.com

Hemisphere

(1) It has six faces and each face is a hexagon

An hemisphere, as the name suggests is half sphere. A sphere cut in half.

(2) It has a hexagonal base with six triangular faces meeting at a point
(3) It has two hexagonal faces and six rectangular faces
(4) It has six hexagonal faces joined by six rectangular faces
Answer: (2) It has a hexagonal base with six triangular faces meeting at a point
Explanation : The keyword here is "pyramid". A pyramid has a polygon base at one
end and an apex at another ( ex cone). An hexagonal pyramid has hexagonal base and
then tapers to a point or an apex.

Q2

Hemisphere
Volume of hemisphere = (2/3)

x r

Where,

is approximately equal to

(22/7) or 3.14
r is radius of sphere
Total surface area = 3 x

x r

Solved geometry problems:


Q1 Which is true for a hexagonal pyramid ?

Ctethelper.blogspot.com

Ctethelper.blogspot.com

=> (h - h2/h) x 100 = (25-16/25) x 100 = 9 x 4 = 36


Answer: (1) 60

=> % height decrement = 36

Explanation: Let consider the front face. It has 5 cubes in one rows and there are three
such rows. Therefore in front face there are ( 5 x 3) 15 cubes. And there are 4 such rows
of (5 x 3) cubes, That's makes a total of 4 x 15 = 60 cubes.
Q4. The internal length, breadth and height of a rectangular box A is 20cm, 18 cm and
15 cm respectively and that of the box B are 18cm, 12cm and 5 cm respectively. The
volume of Box A is how many times that of B?
Q3
(1) 4
(2) 5
(2) 6
(5) 3
Answer:
(1) 5
Solution:

Answer: (3) 36
Solution: Volume of a cylinder = (

x r x h)

Since, diameter and hence radius is increases by 25 % but the volume


remained the same, the height should have been decreased. Let the new height
be h2. And the volume = ( x (1.25 r) x h2). Equating both the

equation, we have,
(

Volume of a rectangular box = height x length x width


Therefor volume of Box A = 20 x 18 x 15 = 5400
And the volume of the box B = 18 x 12 x5 = 1080

Divide the volume of A by the volume of B to get the answer = 5400/1080 = 5

x r x h) = ( x (1.25 r) x h2)

=> h = (25/16)h2
=> (h2/h)= 16/25

Q5. The Whole surface of a cube is 150 sq. cm. Then the volume of the cube is
(a) 125 cm

(b) 216 cm

(c) 343 cm

(d) 512 cm

Ctethelper.blogspot.com

Ctethelper.blogspot.com
(a) 2cm (b) 3cm
(c) 4cm (d) 6 cm

Answer:(a) 125 cm
Solution:

Answer:3 cm
A cube has 6 equal square faces that makes the whole surface area. Let the area of the
square be X sq. cm.

Solution:

Then,

Volume of a sphere = (4/3) r

6 x X = 150
=> X

= 150/6 = 25 sq. cm.

= (4/3)(22/7)(6/2) = (4/3)(22/7)(3)

Again, Volume of the cone = (1/3)(22/7)(12/2)(h) = (1/3)(22/7)(6)(h)

Since, area of the square = l x l =25


=>

l = 5 cm

Since same metal is used in recast the cone from the metal ball
then, the
(4/3)(22/7)(3) = (1/3)(22/7)(6)(h)

And the volume = 5 x 5 x 5 = 125 cm

=>

h = (4)(3)/(6)
= (4)(27)/(36)

Q6. If each edge of a square be doubled, then the increase percentage of its area

= 3 cm

is
(a) 200%

(b) 250%

(c) 280%

(d) 300%

Geometry: Area and Perimeter


1.Triangle :

Answer: (d) 300%


Solution:
Let the edge of the square be X cm then the area = X x X = x
Then if the edge is double then = (2x)(2x) = 4x
Therefore. the percentage increase in area is = {(4x-x)/x} x100 = 300%
Q7.A solid metallic spherical ball of diameter 6 cm is melted and recast into a cone with
a diameter of the base as 12 cm.The height of the cone is

Ctethelper.blogspot.com

Ctethelper.blogspot.com

Area of triangle = (1/2) x b x p


Area of rectangle =

lxb

Where,
where,
b = the length of the side where perpendicular is drawn from opposite

l = length of rectangle
b = breadth of rectangle

point
p = the length of the perpendicular

Perimeter of rectangle = 2 ( l + b )
Perimeter of triangle :

where,

Perimeter of a triangle = (a + b + c),

l = length of rectangle

where,

b = breadth of rectangle

a, b, c are the length of the sides AB, BC and AC.

Diagonal of a rectangle is a line segment joining two opposites vertices of the rectangle.

2. Rectangle : A rectangle is a parallelogram, whose opposite sides are equal and the
angle at which the lines intersect is 90. The diagonals of the rectangle are equal in
length.

Relationship of the diagonal to the sides of rectangle:


______

Ctethelper.blogspot.com
diagonal =l

Ctethelper.blogspot.com

+ b

diagonal =l

+ l = l2

3. Square : A square is a rectangle, whose all sides are equal. Diagonal of a square are
perpendicular bisector of each other.
4. Rhombus: A rhombus is a parallelogram, whose all sides are equal.

Rhombus with side "a" and diagonals d1 and d3

Area of square =

lxl

Area of rhombus =

where l = length of one side

(1/2) x d1 x d2

where d1 = length of shorter diagonal


d2 = length of larger diagonal

Perimeter of square =

2(l+l)=4l

Diagonal of a square is a line segment joining two opposites vertices of the square.
______

__

Perimeter of rhombus =

2(a+a )=4a

Ctethelper.blogspot.com

Ctethelper.blogspot.com

Solved Problems
(2) calculating area of a rectangle by finding length and breadth of a

Q1. The length of a rectangle is l and its width is half of its


length. What will be the perimeter of the rectangle if the
length is doubled keeping the width same ?
(1) 4l
(2) 5l
(3) 6l
(4) 3l

rectangle and using the formula for area of a rectangle (i.e. length x breadth)
(3) calculating area of figures with the help of counting unit square
(4) explaining of formulae for finding area of figures of different shapes

Answer:(1) comparing area of any figure with the help of different objects like palm,
leaf, pencil, notebook, etc.

Q3. A rhombus has diagonals of length 8 cm and 6 cm. Find its perimeter.

Answer: (2) 5l
Solution: Here,for first rectangle length = l and the width = (l/2). A

new rectangle is formed by keeping the width same i.e (l/2)


and doubling the length i.e. 2l .
therefore, perimeter becomes = 2( 2l + l/2)
= 2(5l/2)
= 5l

(1) 18 cm
(2) 20 cm
(3) 24 cm
(4) 28 cm

Answer:(3) 24 cm
Explanation: Area of rhombus = (1/2 )x d1 x d2 ,
where d1 and d2 are the two the diagonal of a rhombus.
here, d1 = 8, d2=6, Therefor, area = (1/2) X 8 X 6 =48/2 = 24

Q2 To introduce the concept of area, a teacher can start with

(1) comparing area of any figure with the help of different objects like palm, leaf, pencil,
notebook, etc.

Q4 .The concept of areas of plane figures can be introduced to the students of Class V
by
(1). calculating the area of a rectangle by finding length and breadth of a rectangle and
using the formula for area of a rectangle

Ctethelper.blogspot.com

Ctethelper.blogspot.com

(2) stating the formula for area of rectangle of rectangle and square
(3). calculating the area of figures with the help of counting unit squares.
(4). measuring the area of any figure with the help of different objects like palm, leaf,
pencil etc.

Answer:(3). calculating the area of figures with the help of counting unit squares.

Q5: The figure consists of five squares of the same size. The area of the figure is 180
square centimeters. The perimeter (in cm) of the figure will be

Explanation: The area of the total figure = 180 sq. cm. Since it is made up of 5 identical
square, the area of one square will come out be = 180/5 = 36 sq.cm. And the length of
the square will work out to be = 36 = 6 cm.
Now, going around the figure we will see, that there are 12 sides in total of al the square
that makes the perimeter of this figure.Hence, the perimeter would come out to be 12 x
6 = 72

Q6

Answer: (4) 4

Q7:
(1) 48
(2) 72
(3) 36
(4) 45

Answer: (2) 72

Ctethelper.blogspot.com

Ctethelper.blogspot.com

3D = 90

=>

=>

D = 90 x (3)

=>

D = 30

Answer: (2) 120

Solution: Since it is given that <d = (1/2) <E. And both angle D and angle E are two
angle of right angled triangle CED. We have,

D + E

+ 90 = 180

Also, it is given that (1/2)A = D

=>
=>

D + 2D

= 180 -90

A = 60 = B ( since CA and CB are equal and so

are angle)

Ctethelper.blogspot.com

And therefore,

BCA

Ctethelper.blogspot.com

= 60

But BCA + ACD = 180 => ACD = 180 - 60 = 120

Solution: In each option, compare the triangles on the left and the right of the
similarity symbol, for similar angles and sides. For example taking option (1),
the angle QRP and similarly taking angle from the other triangle it will be
angle RQT and they are equal. Similarly comparing each sides and angles, we
will confirm that option (1) is correct.

Q9. Perimeter of a square is 24 cm and the length of the


rectangle is 8 cm.If the perimeters of the square and the
rectangle are equal , then the area of the rectangle is

Q8.
(1) 16
(2) 24
(3) 32
(4) 64

Answer: (3) 32
Solution:
Since perimeter of the square and rectangle are equal, then
4S = 2( L + B)
where, 4S = 24, L = 8cm.
Put the values to get
24 = 2(8 + B)
Answer: (1)

PQR TRQ

=>12 = 8 + B
=>4 = B

Ctethelper.blogspot.com

Ctethelper.blogspot.com

Therefore area of the square is = 4 x 8 = 32 square cm

Let say we have to compare two fractions 7/10 and 6/9. To do this comparison either
we take LCM and then make the denominator of each equal to LCM. Or we can simply
convert them to denominator 100 by use of percentage and then we can compare them.

Q10 : The perimeter of the square is 20 cm. A rectangle has the same width as the
square. The length of the rectangle is twice the width. The area in ,square cm, of the
rectangle is :
(1) 30

Now taking 7/10 and converting it percentage, we have (7/10) x 100 = 70 %


and converting 6/9 to percentage , we have (6/9) x 100 = 66.67 %

(2) 50
(3) 100
It is clear now that 7/10 > (6/9) .It is easier than use of LCM to compare them.
(4) 25

Answer: (2) 50

Percentage have a wide area of use from schools to managements decisions, scientific
applications, data interpretation and so on.

Solution:
Perimeter of the square = 20 cm

Percentage Formula

Therefore length of the side of the square = 20/4 = 5cm


Percentage = fraction x 100
A rectangle has the same width as the side of the square. So, width of the rectangle is 5
cm and since it length is double the width, the length comes out to be 10 cm.

the fraction could be any ratio, marked scored against total marks, run scored against
ball faced, energy used against total energy and so on.

Therefore area of rectangle = l x b = 5 x 10 =50 square cm


Converting a decimal to percentage

Percentage
Percentage literally means per Cent or Per 100. In percentage, we convert the fraction,
such that the denominator becomes 100. It is used to compare fraction as we have to
convert all the fractions to a single denominator i.e. 100.

Percentage = Decimal x 100

Expressing a quantity as a percentage on another quantity.

Lets say we have to express x as percentage of another quantity y

Ctethelper.blogspot.com

Ctethelper.blogspot.com
Applying the formula for percentage,

then the formula is (x/y) x 100

Expressing the increase and decrease of a quantity in percentage

If |x| is the absolute increase or decrease in the value of a quantity say 'y' then the
percentage increase or decrease is expressed as

required percentage = 5/4 x 100 = 125 %

3, Half of 1 percent written as a decimal is :


(a)0.005

(b) 0.05

(c) 0.02

(d) 0.2

absolute percentage change = { |x|/y } x 100


Answer: (a)
where the sign of x will determine the increase or the decrease in value.
Solution:
Expressing percentage as a fraction

Let us say the percentage change is x %, then the corresponding fraction is

Fraction = x/100 i.e. just divide the x by 100 to get the corresponding fraction.

Half of 1 percent is 0.5 % = 0.5/100 = .005

(to convert % to decimal you divide it by 100)

4. what is 15 percent of Rs. 34?


(a) Rs. 3.40

(b) Rs. 3.75

(c) Rs. 4.50

(d) Rs. 5.10

1. The ratio 5:4 expressed as percent equals:

(a) 12.5%

(b) 40%

(c) 80%

(d) 125%

Answer:(d)
Solution:

Answer: (d)

Ctethelper.blogspot.com

Ctethelper.blogspot.com

Solution:

Solution :

Applying the formula for conversion of percentage to actual

88 % of 370 = 0.88 x 370 = 325.6

Actual = (x %/100) x N where x is the percentage given and N is the total


Amount.

24 % of 210 = 0.24 x 210 = 50.4


put these terms in the equation and let the unknown be x then we
have,

Here, x = 15 %, N= 34.
Now, put the values to get (15/100) x 34 = 0.15 x 34 = 5.1

325.6 + 50.4 - x = 118


376 - x = 118
x = 258

5. 63% of 3 x (4/7) is :
(a) 2.25

(b) 2.40

(c) 2.50

(d) 2.75

7. 860% of 50 +50% of860 =?


(a) 430

(b) 516

(c) 860

(d) 960

Answer: (a)
Solution:
First convert the mixed fraction 3 (4/7) to fraction which is = (21 + 4)/7 =
25/7.

Answer: (c)

Now, to calculate 63% of 25/7.


Solution:
we have,
860 % of 50 = 8.6 x 50 = 430
( 63/100) x (25/7) = 9/4 = 2.25
50% of 860 = 0.5 x 860 = 430
6. 88% of 370 + 24% of 210 - ? = 118
(a) 256

put the values in the above equation to get 430 + 430 = 860

(b) 258
8. 45% of 750 - 25% of 480 = ?

(c) 268

Answer:(b)

(d) 358
(a) 216

(b) 217.50

(c) 236.50

(d) 245

Ctethelper.blogspot.com

Ctethelper.blogspot.com
(c) 6

(d) none of these

Answer: (b)
Solution: 45 % of 750 = 337.5

Answer:(d)

25 % of 480 = 120

Solution:

Put the values in the above equations to get 337.5 - 120 = 217.5

218% of 1674 = 2.18 x 1674 = 3649.32


Put this value in the above equation to get
3649.72 = x x 1800

9. 40% of 1640 + ? = 35%of 980 +150%of 850


x = 2.0274
(a) 372

(b) 842

(c) 962

(d) 1052

11. 60% of 246 is the same as :


(a) 10% of 44

Answer: (c)

(c) 30% of 132

(b) 15% of 1056


(d) none of these

Solution:
Answer:(d)
40% of 1640 = 656
Solution:
35% of 980 = 343
150% of 850 = 1275

60% 0f 246 = 0.6 x 246 = 147.6

Put these values and x as the unknown value in the equation to get
656 + x = 343 + 1275

or

x = 1618 - 656

Checking the options

x = 962

(a) => 4.4


(b) => 158.4
(c)=> 39.6

10. 218% of 1674 = ? x 1800


(a) 0.5

(b) 4

12. 270 candidates appeared for an examination, of which 252 passed . the
pass percentage

Ctethelper.blogspot.com

Ctethelper.blogspot.com

(a) 80%

(b) 83(1/3) %

(c) 90(1/3) %

(d) 93(1/3)

(c) 18(2/3)%

(d) 22(1/2)%

Answer:(a)
Answer:(d)

Solution:

Solution:

(3/24) x 100 = 12 (1/2) %

Using the formula for calculating percentage we have,


(Total passed/ Total appeared ) x 100 = (252/270 ) x 100 = 93(1/3)
16. It costs Re 1 to photocopy a sheet of paper. However, 2% discount is
allowed on all photocopies done after first 1000 sheets. How much will it
cost to copy 5000 sheets of paper?

13 . 0.01 is what percent of 0.1


(a) (1/100)

(b) (1/10)

(a)Rs. 3920

(b) Rs. 3980

(c) 10

(d) 100

(c)Rs. 4900

(d) Rs. 4920

Answer:(c)

Answer: (d)

Solution:

Solution:

(0.01/0.1) x 100 = (1/10) x 100 = 10 %

Cost of photocopying first 1000 sheets = Rs 1000


Cost of photocopying next 4000 sheets is = 4000 - 0.02 x 4000 = 3920

14 . what percent of Rs. 2650 is Rs 1987.50


Therefore total cost = 4920
(a) 60%

(b) 75%

(c) 80 %

(d) 90%

Answer: (b)

17. A housewife saved Rs. 2.50 in buying an item on sale.If she spents Rs.
25 for the item, approximately how much percent she saved in the
transaction?

Solution:

(a) 8%

(b) 9%

(1987.50/2650) x 100 = 75 %

(c)10%

(d) 11%

15. What percent of a day is 3 hours?

Answer: (b)

(a) 12(1/2)%

Solution:

(b) 16(2/3)%

Ctethelper.blogspot.com

Ctethelper.blogspot.com

Total saved 2.5


Answer : (b)
Total Spent after the discount = 25
Therefore original cost of item = 27.5

Solution: Making it an equation and converting the % into numbers and


unknown into x

And percentage saved = (2.5/27.5)x 100 = 9.09

0.15 x 40 - 0.25 x X = 2
6 - 0.25X = 2
4 = 0.25 X
X = 16

18.The number which exceeds 16% of it by 42 is :


(a)50

(b)52

(c) 58

(d)60

Answer: (d)

20 . The sum of two numbers is 2490. if 6.5% of one number is equal to


8.5% of the other

Solution:

(a)989,1501

Let the number be x then

(c)1401, 1089

(b)1011,1479
(d)1411,1079

{(x - 42) /x} x 100 = 16


Answer: (d)
(x - 42) x 100 = 16x
Solution:
100x - 4200 = 16x
We will have two equations in two variables
84x = 4200
i.e. x + y = 2490
x = 50
65 x =85 y
Solving these equation , we will have y = 1079 and x = 1411
19. If 15% of 40 is greater than 25% of number by 2. then the number is :
(a)12
(c)24

21. The difference of two numbers is 20% of the larger number . if the
smaller number is 20, then the larger number is :

(b)16
(a)25

(b)45

(c)50

(d)80

(d)32

Ctethelper.blogspot.com

Ctethelper.blogspot.com
(a)

Rs. 1200

(b) Rs. 2400

(c)

Rs. 3000

(d) Rs. 3200

Answer: (a)
Solution:
Writing the above condition into equations,

Answer: (b)

( x - y ) = 0.2 x x

Solution:

( x - 20 ) = 0.2 x x

His saving in Percentage is 33 (1/3) % and it is equal to 1200

0.8 x = 20

Therefore, x x (100/3%) = 1200

x = 25

x / 3 = 1200
x = 3600

22. If one number is 80% of the other and 4 times the sum of their squares
is 656, then the numbers are :
(a)4,5

(b)8,10

(c)16,20

(d) None of these

And expense = 3600 - 1200 = 2400

Answer:(b)

24. 10% of of the voters did not cast their in an election between two
candidates. 10% of the votes polled were found invalid;id.The successful
candidate got 54% of the valid votes and won by a majority of 1620 votes.
The number of voters enrolled on the voters' list was:

Solution:

(a) 25000

Here we can check for options or form two equations in two variable to get
the answer.

(c) 35000

We will follow the first approach

Answer:(a)

We can observe that the first condition is satisfied by the first three options.

Solution:

(b) 33000
(d) 40000

Now check for second condition


(a) 4 ( 16 +25) = 4 (41) = 164

Not satisfied

First we calculate the valid votes, They are divided into two candidates and
the winning candidate got 54 % and losing got 46 %. the difference is 8 %
and it is equal to 1620.

(b) 4 ( 64 + 100) 4 (164) = 656 satisfied. Hence this is the answer.


i.e. 0.08 x x = 1620
23. A person who spends 66(2/3)% of his income is able to save Rs. 1200
per month. His monthly expenses ( in Rs. ) are :

x = 20250 Votes that were valid


and since 10 % votes were invalid.

Ctethelper.blogspot.com

Ctethelper.blogspot.com

0.9 x vote polled = 20250

(a)58%

(b) 62(2/3) %

(c)64%

(d) 78%

Votes polled = 22500


Answer: (b)
And Since 10 % did not cast their votes
Solution:
i.e.e0.9 x Total Voters = 22500
42% boys passed that means 58 boys failed which is equal to

= 638

Total Voters = 25000


30% girls passed that means 70 % girls failed which is equal to = 490
Total Appeared = 1800
25. Two tailors X and Y are paid a total of Rs. 550 per week by their
employer. If X is paid 120 percent of the sum paid to Y, how much is Y paid
per week?

Total Failed % = (1128/1800) x 100 = 62 (2/3)

(a)

Rs. 200

(b) Rs. 250

27 . If 20% of a= b, then b% of 20 is the same as :

(c)

Rs. 300

(d) None of them

(a) 40%o of a

(b)5% of a

(c)20% of a

(d)None of these

Answer: (b)
Solution:

Answer: (a)

Total Paid to tailor in 1 week = 550

Solution:

Let Y is Paid at the rate of P per week

Given ,

Then X is paid at the rate of 1.2 P per week

0.2 x a = b

And this total comes to be 550 = 1P + 1.2P

Then ,

2.2P = 550

b% X 20 = (b/100) x 20 = {(0.2 x a)/100} x 20 = 0.4 x a or 40 percent of a

P= 250
29 If p% of p is 36, then p is equal to :

26 . 1100 boys and 700 girls are examined in a test , 42 % of the boys and
30 % of the girls pass . the percentage of the total who failed is :

(a) 15

(b)60

(c) 600

(d)3600

Ctethelper.blogspot.com

Ctethelper.blogspot.com

Answer: (b)
31. A man loses 20(1/2) % of his money and after spending 80% of the
Solution:
remainder, he is left with 159. How much money did he have first?
Given
(a) 800

(b) 1000

(p/100) x p = 36
(c) 1,200 (d)

500

P X p = 3600
p = 60
Answer: (b) 1000
30. If x % of y is 100 and y% of z is 200 then find a relation between x and
z

Solution:
Let the money was Rs x .Then after losing 20.5 % he is left with (0.795)x

(a)z= (x/ 2)

(b) z = 2 x

(c) z= (x/4)

(d)z=4 x

Now, he spend 80% of the remainder and left with 159.


This means 20% of 0.795x is 159.
Answer: (a)

=>

(0.795x)(0.20) = 159

Solution:

=>

0.159x = 159

First

=>

x = 159/0.159 = 1000

(x/100) y = 100
(xy) = 10000 --------- (i)

32. If 120% of a is equal to 80% of b, then (b+a)/(b-a) is equal to


(a) 5 (b) 6
(c) 7 (d) 8

And (y/100)z = 200


(yz)=20000------- (ii)
Answer:(a) 5
Dividing Equation (i) by equation (ii) we get
(x/z) = (1/2)
2x = z
z = (x/2)

Solution:
Given that 120% of a = 80% of b,
=> 1.20 x a = (0.80)b

Ctethelper.blogspot.com
=> a = (8/12)b
=> a = (2/3)b

Thus, (b+a)/(b-a) = {b+(2/3)b }/{b - (2/3)b}


= (5/3)/(1/3) = 5

Das könnte Ihnen auch gefallen